OSCE Stations Checklists in History and Examinations

You might also like

Download as pdf or txt
Download as pdf or txt
You are on page 1of 94

1 HEAD & NECK OSCE ‐ Stasions

HEAD & NECK STATIONS:

1. Posterior Neck Mass (**) → History and Examination.


2. Neck Mass → History.
3. Thyroid → Exam.
4. Thyroid → Exam, Investigations & Diagnosis.
5. Neck → Exam, Investigation & Diagnosis (retrosternal Goiter).

NECK MASS / LUMP (HISTORY)

Greet & Introduce yourself. Ask his/her permission


Begin by asking: Name, Age, Occupation, Chief complaint (what has caused you to come
and see me today)… if not mentioned previously (in the case scenario).

POINTS TO BE ASKED
Age:
Children infections (usually), cystic hygroma, congenital dermoid cyst.
1
Young adult Branchial cyst, carotid body tumour (˃ 30 Y/O)
Older Adults pharyngeal pouch (˃ 50 y/o), malignant LN (pharyngeal/laryngeal CA).
Work:
2 Those worked abroad TB (‫)عملت خارج البلد‬, health worker (radiation) CA
Asbestose, nickel exposure [industrial plant (alloys, batteries)] Laryngo-Pharyngeal CA.
Duration (onset)
3 < 3 wk LAP.
Since birth Congenital → if yes ask if diagnosed intra-nataly by U/S.
4 First Time / Recurrent (Previous Similar condition).
Number:
5 How many lumps have you noticed (Single)/ (multiple LN, Lipomata, Sebaceous cysts).
6 Unilateral or (bilateral Parotitis, Mumps, Sjogren/mikulicz syndrome).
7 any other site lump(s) →axilla, groin, abdomen (LN metastasis)
Size change:
8 Any change in size (smaller, bigger or remaining the same).
9 Continuous or intermittent Salivary stone →Ask about precipitate factors like food.
Pharyngeal pouch → size (bulge) on swallowing.
10 Color change melanoma
11 Trauma / Insect Bite [pyogenic granuloma; after minor trauma / bite].
Painful or painless (if yes → pain features → severity, radiation, precipitating & alleviating
12
factors) [e.g. Salivary gland stone] any other region Pain (Neck, Chest, Abdomen)
13 Temperature change / sweeting at night
14 Travelling Infectious (recent travel), EBV (China/Hong Kong)
ENT:
15 Dental problem dental sepsis
16 Voice Change (hoarseness) Bronchus, Esophagus, Thyroid (all with/without LN
metastasis).
17 Sore throat (‫)احتقان في الحلق‬ Pharyngeal pouch, oropharyngeal CA.
18 Halitosis (‫)رائحة فم كريھة‬ Pharyngeal pouch.
19 Otalgia (ear pain) Pharyngeal/Laryngeal CA → Hoarse voice + Painful ear.

POST‐GRADUATE GENERAL SURGERY OSCE (By Ali M J) 1


1 HEAD & NECK OSCE ‐ Stasions

Respiratory:
20 Difficult breath.
21 Cough (dry or productive).
22 Hemoptysis (Blood) CA, infection (TB).
23 Chest infections (single or frequent) Pharyngeal pouch, CA.

24 Breast →any lump or nipple discharge


GIT:
25 Odynophagia (Painful Swallowing) Pharyngeal pouch.
26 Dysphagia (difficult Swallowing) Pharyngeal pouch, CA continue with dysphagia
Hx.
27 Bulge ( size/ ‫ )انتفاخ‬or gurgle (‫صوت‬/‫ )قرقرة‬on swallowing Pharyngeal pouch.
28 Nausea, Vomiting, early satiety Virchow’s Node (Gastric CA)
29 Hematemesis & Malena.
30 Weight loss.
Past medical:
Chronic disease (DM, HT, HF, liver diseases).
31 Drugs (regular medications), or medication for this problem
Allergies.
Previous hospital admission, investigation (especially U/S, FNA).
Past surgical:
Previous operation.
32
Radiotherapy.
Chemotherapy.
Family Social History:
33 Similar family condition, Family history of malignancies.
Smoking & Alcohol.
THANKING the patient.

POST‐GRADUATE GENERAL SURGERY OSCE (By Ali M J) 2


1 HEAD & NECK OSCE ‐ Stasions

NECK MASS / LUMP (EXAMINATION)

Great and Introduce yourself to the patient, and ask his/her permission.
Positioning and exposure.
Wash/Gel your hands.
Ask (Is it Painful or not? Any other pain region?).

SPECIFIC LUMP EXAM


INSPECTION
1 general signs (Myxedema, Hyperthyroidism or Plethoric face (SVC obstruction)
2 Site (Midline, Supraclvicular fossa, Lateral) (Neck triangle)
3 Size (if small; roughly) (if large; tape)
4 Shape
5 Skin changes (over the lump)
6 Surrounding skin
7 Symmetry (unilateral, or bilateral)
8 Surface (Smooth, nodular).
9 Scar, Color change
special characteristics:
10 pulsatility.
movement with swallowing & tongue protrusion.
PALPATION
11 Tenderness (ask patient)
12 Temperature (dorsum of hand)
13 Consistency (Firm, soft, spongy)
14 surface (smooth, irregular)
15 Edge (well/poorly defined)
Fixation (skin or muscles)
• Move lump in two directions, right-angled to each other. Then repeat exam when muscle
contracted:
• Bone: immobile.
16
• Muscle: contraction reduces lump mobility.
• Subcutaneous: skin can move over lump.
• Skin: moves with skin.

Compressibility (disappear by pressure an reappear directly)


17
Reducibility (disappear by pressure and not reappear unless other force like cough)
Pulsation (place one finger on each opposite site of the lump)
18 • Transmitted pulsation: both fingers pushed same direction.
• Expansile: fingers diverge (esp for AAA).
19 Fluctuation (one hand 2 finger and other hand index finger) "paget’s sign" / "peace sign"
20 Fluid thrill
21 AUSCULTATION → Bruit.

POST‐GRADUATE GENERAL SURGERY OSCE (By Ali M J) 3


1 HEAD & NECK OSCE ‐ Stasions

ASOCIATETED EXAM
22 Palpate Carotid A
23 Thyroid gland anteriorly
24 Cervical lymph nodes ( all groups) from behind
Mouth examination:
Tongue.
Teeth.
25 Floor of the mouth.
Waldeyer's tonsillar ring.
Parotid Duct Orifice.
Bimanual examination.
I Finished My Exam with:
1. Other lymphoid tissues:
Axilla.
26 Groin.
2. Abdomen for HSM.
3. ENT exam (laryngoscopy, bronchoscopy)
4. Chest examination
THANKING the patient.

POST‐GRADUATE GENERAL SURGERY OSCE (By Ali M J) 4


1 HEAD & NECK OSCE ‐ Stasions

CERVICAL LN (EXAMINATION)

Great and Introduce yourself to the patient, and ask his/her permission.
Positioning and exposure.
Wash/Gel your hands.
Ask (Is it Painful or not? Any other pain region?).

INSPECTION
1 General look, Swelling , Scars ,Dilated veins
2 Asking the patient to swallow
PALPATION
3 Palpate Carotid A
4 Thyroid gland anteriorly
5 Cervical lymph nodes ( all groups) from behind
Mouth examination:
6 Tongue.
7 Teeth.
8 Floor of the mouth.
9 Waldeyer's tonsillar ring.
10 Parotid Duct Orifice.
11 Bimanual examination.
I Finished My Exam with:
12 1. Other lymphoid tissues:
Axilla.
Groin.
13 2. Abdomen for HSM.
14 3. ENT exam (laryngoscopy, bronchoscopy)
15 4. Chest examination
THANKING the patient.

POST‐GRADUATE GENERAL SURGERY OSCE (By Ali M J) 5


1 HEAD & NECK OSCE ‐ Stasions

INVESTIGATION OF (NECK LUMP)

1. Chest X-ray:
Chest radiology is important in young adults, when lymphoma is the possibility.
TB
Primary or secondary lung neoplasms.

2. Ultrasound
Can differentiate between solid and cystic masses
Can indicate whether or not there are multiple enlarged lymph nodes or the presence of
multiple nodules in the thyroid gland.
Ultrasound, however, rarely assists in clarifying the diagnosis.

3. FNA (Fine-needle aspiration biopsy):


The single most important test in the evaluation of neck lumps, particularly in adults who
may have malignancy
Contraindicated in carotid body tumor.
Painful in schwanoma (tinnel’s test may be positive)

4. Excision biopsy:
If a diagnosis cannot be confirmed on fine-needle aspiration biopsy, an excision biopsy may
be necessary to confirm or exclude malignancy.

5. Computed tomography (CT) scans / MRI:


Are far more helpful than ultrasound in assisting with the diagnosis of neck swellings, especially
when they are larger than 2 cm → consistency, size, anatomical relations.

6. Angiography
The gold standard in carotid body tumor (Lyre’s Sign).

7. Barium swallow:
In pharyngeal pouch.

POST‐GRADUATE GENERAL SURGERY OSCE (By Ali M J) 6


1 HEAD & NECK OSCE ‐ Stasions

NECK MASS DIFFERENTIAL DIAGNOSIS

Lateral Structures
Superficial MIDLINE
ANTERIOR POSTERIOR
- - Submental LN - Lymph Node (levels II, - Lymph Node (level V and
Sebaceous - Sublingual dermoid III, IV). supraclavicular lymph node
cyst cyst - Submandibular Gland groups)
- Abscess - Thyroglossal cyst mass - Cervical rib
- Lipoma - Plunging ranula - Carotid body Tumor - Brachial plexus
- Dermoid (retention cyst of the (Chemodectoma) or neuroma/schwannoma
sublingual) Aneurysm. - Cystic Hygroma.
- Rarely, hyoid pathology - Branchial cyst (+ - Aberrant thyroid (2ry
e.g. bursa Fistula) deposit of Papillary thyroid
- Thyroid nodule in the - Cold Abscess (TB) CA).
isthmus - Thyroid Nodule. - Subclavian A aneurysm.
- Pharyngeal pouch
(Zenker’s
diverticulum)
- Laryngocoele (rare; an
air-filled, compressible
structure seen in glass-
blowers)

Also DDx according to:


Cystic or solid
Single or multiple.
Inflamatory
Neoplastic
Conginetal

Notes:
If SCC → Panendoscopy
If Adenocarcinoma → Breast Stomach, Pharynx
Lymph node → Biopsy.

POST‐GRADUATE GENERAL SURGERY OSCE (By Ali M J) 7


1 HEAD & NECK OSCE ‐ Stasions

THYROID (HISTORY)

Greet & Introduce yourself. Ask his/her permission


Begin by asking: Name, Age, Occupation, if not mentioned previously.

A. Local Symptoms (from the swelling)


1 Duration
change in size (it’s gradual or sudden onset increased size; hemorrhage or anaplastic
2
carcinoma)
3 Pain (Thyroiditis, anaplastic carcinoma)
4 Discomfort during swallowing/dysphagia - esophageal compression
5 Breathe discomfort/ Dysphonia/ Strider (tracheal compression).
6 Hoarseness (recurrent laryngeal nerve; malignancy).
7 Can be seen? cosmatic
A. Thyroid status
Mood change (anxiety, sleep disturbance; hyperthyroidism) (depression, fatigue ;
8
hypothyroidism)
9 Vision (Graves; double vision)
Appearance:
Thinning hair.
10
Dry or sweating skin.
Loss outer third eyebrow.
11 Intolerance to hot (hypothyroidism) or cold (hyperthyroidism)
12 Palpitation, chest pain.
13 Appetite (increased in hyperthyroidism)
14 Weight change (gain; hypothyroidism)(loss; hyperthyroidism)
15 Bowel habit (Constipation; hypo.) (Diarrhoea; hyper.)
16 Period change (Female) (Menstrual disturbances) amenorrhoea, Menorrhagia
17 Miscarriages and infertility (Female)
18 Proximal myopathy (autoimmune) with wasting [Hyperth.], Muscle fatigue [Hypoth.]
19 Fine tremor (hyperthyroidism) hand and wrist pain (carpal tunnel; hypothyroid)
Past medical:
Chronic disease (DM, HT, HF, liver diseases).
20 Drugs (regular medications), or medication for this problem (antithyroid drugs)
Allergies.
Previous hospital admission, investigation (especially U/S, FNA).
Past surgical:
Previous operation.
21
Radiotherapy.
Chemotherapy.
Family Social History:
22 Similar family condition, Family history of malignancies.
Smoking & Alcohol.
THANKING the patient.

POST‐GRADUATE GENERAL SURGERY OSCE (By Ali M J) 8


1 HEAD & NECK OSCE ‐ Stasions

THYROID (EXAMINATION)

Great and Introduce yourself to the patient, and ask his/her permission.
Positioning and exposure.
Wash/Gel your hands.
Ask (Is it Painful or not? Any other pain region?).

INSPECTION (from front and side of the patient)


Swelling, scars, dilated veins, visible jugular pulse or plethoric face (SVC obstruction).
Ask to open month (look for lingual thyroid)
Ask the patient to protrude tongue (Thyroglossal cyst)
Ask the patient to swallows (sip of water)
Arm raising test for retrosternal goiter (pemberton sign)
PALPATION (from FRONT)
Thyroid Temperature.
Thyroid gland anteriorly (confirming the visual swelling and tenderness)
position of the trachea
Carotid pulse (absence = Berry’s sign for malignant)
PALPATION (from BEHIND)
Thyroid gland posteriorly
Ask the patient to protrude tongue (Thyroglossal cyst).
Ask the patient to swallows.
Cervical lymph nodes
PERCUSSION
Clavicle and manuberium (for retrosternal goiter)
AUSCULTATION
Over the upper pole of thyroid gland for bruit
wheeze in the upper respiratory tract suggestive of obstruction from the thyroid mass
Thyroid status
HANDS SIGNS
feel the hands for:
skin change (discolouration, pigmentation, vitiligo).
Sweating.
Palmar erythema [hyperthyroidism].
Nails for Thyroid acropachy & onycholysis (plummer’s nail).
Finger acropatchy (pseudoclubbing) [Graves].
Tremor – placing a piece of paper on the backs of the patient’s outstretched hands.
Radial Pulse (It may be tachy or bradycardic)

POST‐GRADUATE GENERAL SURGERY OSCE (By Ali M J) 9


1 HEAD & NECK OSCE ‐ Stasions

EYE SIGNS
Joffroy's sing: absence of forehead wrinkling on eye brows elevation, thyrotoxicosis.
Hair loss of lateral third of the eyebrow [Hypoth].
Stellwag's sign: infrequent & incomplete blinking
Chemosis [hyperth].
Exophthalmos (sclera visible all around the iris) (normally only 1/5).
Proptosis (Front, Lteral & Behind the patient).
Lid retraction (Dalrymple’s sign)
lid lag (Von Graefe's sign)
opthalmoplegia (ocular movement; superior recti & inf oblique) leads to diplopia when
looking up and out
LEGS SIGNS
Pretibial myxoedema
Reflexes (ankle reflex delayed in hypothyroidism)
proximal myopathy (ask the patient to stand up with arms across the chest).
FINISHING
Vocal cord by flexible laryngoscopy.
THANKING the patient

POST‐GRADUATE GENERAL SURGERY OSCE (By Ali M J) 10


1 HEAD & NECK OSCE ‐ Stasions

INVESTIGATION OF PATIENTS WITH (THYROID DISEASE):

A. Blood Test of thyroid function:


Serum Thyroid-Stimulating Hormone (TSH)
Thyrotropin-Releasing Hormone
This test is useful to evaluate pituitary TSH secretory function and is performed by administering
500 g of TRH intravenously and measuring TSH levels after 30 and 60 minutes. In a normal
individual, TSH levels should increase at least 6 IU/mL from the baseline.
Total T4 (Reference Range 55–150 nmol/L) and T3 (Reference Range 1.5–3.5 nmol/L).
Total T4 and T3 levels are measured by radioimmunoassay and measure both the free and bound
components of the hormones
Thyroid Antibodies
Thyroid antibodies include anti-Tg, antimicrosomal, or anti-TPO and thyroid-stimulating
immunoglobulin (TSI). About 80% of patients with Hashimoto's thyroiditis have elevated thyroid
antibody levels; however, levels may also be increased in patients with Graves' disease, multinodular
goiter, and occasionally, with thyroid neoplasms.

Serum Thyroglobulin
Tg is only made by normal or abnormal thyroid tissue. It normally is not released into the circulation
in large amounts but increases dramatically in destructive processes of the thyroid gland, such as
thyroiditis, or overactive states such as Graves' disease and toxic multinodular goiter. The most
important use for serum Tg levels is in monitoring patients with differentiated thyroid cancer for
recurrence, particularly after total thyroidectomy and RAI ablation.

B. Ultrasound
o An excellent noninvasive
o Distinguishing solid from cystic ones.
o Providing information about size and multicentricity.
o Cervical lymphadenopathy.
o To guide FNAB. An experienced ultrasonographer is necessary for the best results

C. FNA
These incidentally discovered nodules should be worked up by ultrasound and fine-needle aspiration
biopsy (FNAB).

D. Chest X-ray

E. Radionuclide Imaging
Both iodine 123 (123I) and iodine 131 (131I) are used to image the thyroid gland. The former emits
low-dose radiation, has a half-life of 12 to 14 hours, and is used to image lingual thyroids or goiters.
In contrast, 131I has a half-life of 8 to 10 days and leads to higher-dose radiation exposure.
Therefore, this isotope is used to screen and treat patients with differentiated thyroid cancers for
metastatic disease.

F. Computed Tomography/Magnetic Resonance Imaging Scan


useful in evaluating the extent of large, fixed, or substernal goiters (which cannot be evaluated by
ultrasound) and their relationship to the airway and vascular structures. Noncontrast CT scans should
be obtained for patients who are likely to require subsequent RAI therapy. If contrast is necessary,
therapy needs to be delayed by several months. Combined PET-CT scans are increasingly being used
for Tg-positive, radioactive iodine–negative tumors.

POST‐GRADUATE GENERAL SURGERY OSCE (By Ali M J) 11


1 PAROTID GLAND-FACIAL NERVE OSCE ‐ Stasions

PAROTID GLAND/FACIAL NERVE (EXAMINATION)

Great and Introduce yourself to the patient, and ask his/her permission.
Positioning and exposure.
Wash/Gel your hands.
Ask (Is it Painful or not? Any other pain region?).

1 Inspection (Site, size, shape, symmetry, skin change, scar, color)

Palpation (Tenderness, temperature, surface, edge, consistency, fixation, compressibility,


2
pulsation, fluctuation, fluid thrill & Transillumination)
Lymph node exam in orders (sbmental, submandibular, parotid, pre-auricular, post-
3 auricular, ant. Triangle, supraclavicular, infraclavicular, post. Triangle node and occipital
nodes).
Examine inside of mouth (using torch) (wearing gloves)
Inspection (ulcer, tonsillar abscess)
4 Palpation (Stone in parotid duct; 2nd upper molar tooth)
(palpate the parotid by finger inside, other hand ………………….….finger
outside below angle of mandible)
Facial nerve exam (as below)
Inspection:
facial expression
slow eyelid closure of affected side
5
widened palpebral fissure
flattened nasolabial fold
mouth droops with less movement (affected side)
MOTOR (Muscles) tests:
Raise eyebrows (occipitofrontalis).
6 Close eye (orbicularis oculi).
Show teeth (orbicularis oris).
Puff out cheek (Buccinator).
Sensory (taste):
7 Anterior 2/3 of the tongue on both sides taste for sweet/salt/bitter /sour (chorda tympani
nerve function)
Secretary function (tear secretion)
8 Scirmer's test (piece of tissue paper under the lower eyelid and removed after 5 minutes to test
the tear secretion)

External auditory meatus (herpes zoster; Ramsay hunt syndrome)


9
Hearing test [stapedius muscle]

10 Complete full ear, nose & throat exam


Thanking patient

POST‐GRADUATE GENERAL SURGERY OSCE (By Ali M J) 1


2 BREAST OSCE ‐ Stasions

BREAST:

1. Nipple Discharge (red color) → History & Investigation.


2. Breast Exam → 2 photo (what is the mistake in the eaxam)
3. Cushing → History & Investigations.

NIPPLE DISCHARGE (HISTORY)

Greet & Introduce yourself. Ask his/her permission


Begin by asking: Name, Age, Occupation if not mentioned previously (in the case scenario).

1 Age (˃40 year risk group)


2 Duration
3 Unilateral or Bilateral
4 Single duct or multiple duct involve.
5 Intermittent or persist
Spontaneously? ("Does discharge stain underclothing or bed clothing?" or occurs only with
6
nipple stimulation or Exercise?
7 Recurrent (or first time)
8 Color (bloody, serous, milky, yellow to brown, purulent).
9 Any associated mass?
10 Pain?
11 Recent emotional stress.
Review of systems should seek symptoms suggesting possible causes, including:
12 Fever (mastitis or breast abscess).
13 Cold intolerance, constipation, or weight gain (hypothyroidism).
14 Amenorrhea, infertility, headache, or visual disturbances (pituitary tumor).
15 Ascites or jaundice (liver disorders).
Decreased libido.
16 Recent pregnancies, lactation, abortions.
(Galactorrhea is normal during pregnancy and for up to one year after cessation of Lactation)
Past medical:
Chronic disease (DM, HT, HF, RF, thyroid, liver diseases).
Drugs (regular medications H2-antagonists (eg, cimetidine), opioids, and tricyclic
17 antidepressants), or medication for this problem
Allergies.
Previous hospital admission, investigation (especially U/S, FNA).
Past surgical:
Previous operation.
18 Radiotherapy.
Chemotherapy.
Family Social History:
Similar family condition, Family history of malignancies. Multiple endocrine neoplasia (esp.
19 Type I)
Smoking & Alcohol.
THANKING the patient.

POST‐GRADUATE GENERAL SURGERY OSCE (By Ali M J) 1


2 BREAST OSCE ‐ Stasions

NIPPLE DISCHARGE (INVESTIGATION):

A. LAB Tests:
1. Urine Pregnancy Test (Quantitative Serum Beta hCG).
2. Serum Prolactin Level (Delay measurement until at least 30 minutes or more after vigorous
Exercise or Breast Exam or stimulation)
3. Thyroid Stimulating Hormone (TSH) Level.
4. Serum Creatinine.
5. Sex hormones (if Hypogonadism suspected) Serum Estrogen, Testosterone, FSH & LH.

B. Guaiac Test (Blood occult test)


C. Cytology.
D. Ultrasonography (If there is a palpable mass, evaluation as for breast mass is done, usually
beginning with US and then FNA)
Lesions that appear cystic are sometimes aspirated, and solid masses or any that remain after
aspiration are evaluated with mammography followed by imaging-guided biopsy.
E. Mammography: If there is no mass but cancer is otherwise suspected or if other tests are
indetermina
F. ductography [Galactogram (Ductogram)]: contrast-enhanced imaging of the milk duct If
mammography and ultrasonography do not identify a source and the discharge is spontaneous
and comes from a single duct or breast.
G. Ductoscopy:

(NOTE) Culture and sensitivity are not useful. Usually grows skin contaminant

CAUSES OF DISCHARGES FROM THE NIPPLE


B. Discharge from more than one duct
A. Discharge from the surface 1. Blood-stained
Paget’s disease Carcinoma
Skin diseases (eczema, psoriasis) Ectasia
Rare causes (e.g. chancre) Fibrocystic disease

2. Serous
Fibrocystic disease
C. Discharge from a single duct Duct ectasia
1. Blood-stained Carcinoma
Intraduct papilloma
Intraduct carcinoma 3. Black or green
Duct ectasia Duct ectasia

2. Serous (any colour) 4. Purulent


Fibrocystic disease Infection
Duct ectasia
Carcinoma 5. Milk:
Lactation
Rare causes (hypothyroidism, pituitary
tumour)

POST‐GRADUATE GENERAL SURGERY OSCE (By Ali M J) 2


2 BREAST OSCE ‐ Stasions

BREAST LUMP / MASS (HISTORY)

Greet & Introduce yourself. Ask his/her permission


Begin by asking: Name, Age, Occupation if not mentioned previously (in the case scenario).

POINTS TO BE ASKED
1 Age.
2 Duration.
3 First Time / Recurrent (Previous Similar condition).
4 Site.
5 Single / multiple.
6 Unilateral / Bilateral.
7 Any other site lump(s) →axilla, groin, abdomen (LN metastasis).
8 Change size (bigger, smaller, same size) is it variation with menstrual cycle?
9 Continuous or intermittent.
10 pain (cyclical or not)
11 Breast size & shape change Skin.
12 Nipple change or Discharge.
13 Arm swelling
14 Hx of Trauma (Cyst or Fat necrosis).
15 Fever
16 Weight loss
17 Jaundice, Bone pain, Breathing problems Metastasis.
18 Breast feeding
19 Contraceptive pills
20 Menstrual history (menarche, menopause) & Childrens
Past medical:
Chronic disease (DM, HT, HF, RF, thyroid, liver diseases).
21 Drugs (regular medications) or medication for this problem
Allergies.
Previous hospital admission, investigation (especially U/S, FNA).
Past surgical:
Previous operation.
22
Radiotherapy.
Chemotherapy.
Family Social History:
23 Similar family condition, Family history of malignancies (Breast & Ovaries).
Smoking & Alcohol.
THANKING the patient.

POST‐GRADUATE GENERAL SURGERY OSCE (By Ali M J) 3


2 BREAST OSCE ‐ Stasions

BREAST (EXAMINATION)

Great and Introduce yourself to the patient, and ask his/her permission.
Positioning and exposure 45 degree.
Wash/Gel your hands.
Ask (Is it Painful or not? Any other pain region?).

INSPECTION (From foot of bed & patient right side)


Exposed from the waist up and lie at 45° on the couch
General inspection for JACCOL + Cachexia
Asymmetry of size, shape, skin changes (redness, retraction, puckering, Peau d’orange),
nodule, ulcer, dilated veins
Nipple & areola (crack, fissure, eczema) (level, retraction, asymmetry, discharge)
Auchincloss’s method (slowly raise both arms over head) & then Hands behind head
Assess fixation of mass, puckering more evident.
Under the breast.
Axilla (swelling LN., veins, muscle wasting)
Hand on hips & press in (tense pectorals & accentuates asymmetry & tethering)
Forward lean (accentuates abnormality in large pendulous breast)
Inspect abdomen (TRAM flap)
Inspect the back (for latissimus dorsi reconstruction)
Surgical sequelae after mastectomy like scapula winging
PALPATION beginning with unaffected side (redress other breast)
tenderness
Temperature
Hand behind head & tilt to contralateral side (breast will lie flat on the chest)
Palpate 7 areas (4 quadrants, axillary tail, nipple areolar complex,
inframammary fold)
The areola & nipple between 2 finger (consistency)
Nipple discharge (squeeze each quadrant toward nipple)
Examine the breast with lump
Lump description (site; each quadrant, shape, size; cm, surface; smooth or irregular,
edge; well-defined & consistency; soft, firm, hard)
Fluctuation.
Transillumination (dark room, torch)
Assess fixation of lump:
Skin (puckering by inspection attach to skin, bet 2 finger tethering above lump; not
attach to skin)
To breast tissue (2 fingers & 2 hands).
To muscles [patient hands on hip] move the lump with pectoralis muscle direction and
then in right angle [with & without tension]. If immobile with relax muscles [hands on
hips without pushing] it’s fixed to the chest wall (when you move the lump, whole chest
move).
Push the wall (serrates anterior) palpate breast and inspect for winged scapula (long
thoracic N.)

POST‐GRADUATE GENERAL SURGERY OSCE (By Ali M J) 4


2 BREAST OSCE ‐ Stasions

Feel axilla of both sides [non palpated hand support elbow at shoulder level]
Lateral, medial, anterior, posterior & apex
Supraclavicular fossa (both sides)
Complete:
chest [Percussion & auscultation to(effusion, consolidation)].
Abdomen palpation for hepatomegaly.
Axil spine Percussion for tenderness.
THANKING the patient.

POST‐GRADUATE GENERAL SURGERY OSCE (By Ali M J) 5


2 BREAST OSCE ‐ Stasions

Clinical Classification of Gynecomastia


Grade I Mild breast enlargement without skin redundancy
Grade IIa Moderate breast enlargement without skin redundancy
Grade IIb Moderate breast enlargement with skin redundancy
Grade III Marked breast enlargement with skin redundancy and ptosis, which simulates a
female breast

Gynecomastia Causes:
1) Physiological
2) Pathological
3) Drug-induced
4) Idiopathic - especially in the elderly

1) physiological:
Newborns
Adolescence or puberty associated
Elderly

2) pathological:
A) Increased oestrogen production:
Increased availability of androstenedione for extraglandular conversion, for example:
liver disease especially cirrhosis - reduced hepatic extraction of androstenedione
thyrotoxicosis - gynaecomastia occurs in about one third of men with thyrotoxicosis
adrenal carcinoma - also, rarely, adrenal tumours may secrete oestrogen directly
congenital adrenal hyperplasia
starvation and refeeding - by a similar mechanism to that in liver disease

due to increased testicular secretion


testicular tumours: stromal cell tumours ie. Leydig and Sertoli cell
bronchogenic carcinoma
true hermaphroditism

B) Reduced testosterone production:


Klinefelter's syndrome.
viral orchitis - especially, mumps
trauma - the second most common acquired cause in adults
castration
renal failure
neurological diseases such as spinal cord lesions
leprosy

3) Drug-induced:
ketoconazole, metronidazole, spironolactone

POST‐GRADUATE GENERAL SURGERY OSCE (By Ali M J) 6


2 BREAST OSCE ‐ Stasions

Classification of Benign Breast Disorders


Nonproliferative disorders of the breast
Cysts and apocrine metaplasia
Duct ectasia
Mild ductal epithelial hyperplasia
Calcifications
Fibroadenoma and related lesions
Proliferative breast disorders without atypia
Sclerosing adenosis
Radial and complex sclerosing lesions
Ductal epithelial hyperplasia
Intraductal papillomas
Atypical proliferative lesions
Atypical lobular hyperplasia
Atypical ductal hyperplasia

Treatment of Recurrent Subareolar Sepsis


Suitable for Fistulectomy Suitable for Total Duct Excision
Small abscess localized to one segment Large abscess affecting >50% of the areolar
circumference
Recurrence involving the same segment Recurrence involving a different segment
Mild or no nipple inversion Marked nipple inversion
Patient unconcerned about nipple inversion Patient requests correction of nipple inversion
Younger patient Older patient
No discharge from other ducts Purulent discharge from other ducts
No prior fistulectomy Recurrence after fistulectomy

Gail Model
1) Age at menarche (years)
2) Number of biopsies/history of benign breast disease, age <50 y and those ˃ 50 y
3) Age at first live birth (years)
4) Number of first-degree relatives with history of breast cancer

Risk management strategies for BRCA1 and BRCA2 mutation carriers include the following:

1. Prophylactic mastectomy and reconstruction


2. Prophylactic oophorectomy and hormone replacement therapy
3. Intensive surveillance for breast and ovarian cancer
4. Chemoprevention

POST‐GRADUATE GENERAL SURGERY OSCE (By Ali M J) 7


2 BREAST OSCE ‐ Stasions

Classification of Breast Ductal Carcinoma In Situ (DCIS)


1) Comedo
2) Intermediate (Often a mixture of noncomedo patterns)
3) Noncomedo (Solid, cribriform, papillary, or focal micropapillary)

Classification of Benign Breast Disorders


Nonproliferative disorders of the breast

Cysts and apocrine metaplasia


Duct ectasia
Mild ductal epithelial hyperplasia
Calcifications
Fibroadenoma and related lesions
Proliferative breast disorders without atypia
Sclerosing adenosis
Radial and complex sclerosing lesions
Ductal epithelial hyperplasia
Intraductal papillomas
Atypical proliferative lesions
Atypical lobular hyperplasia
Atypical ductal hyperplasia

Foote and Stewart originally proposed the following classification for invasive breast cancer:
1. Paget's disease of the nipple
2. Invasive ductal carcinoma
3. Adenocarcinoma with productive fibrosis (scirrhous, simplex, NST), 80%
4. Medullary carcinoma, 4%
5. Mucinous (colloid) carcinoma, 2%
6. Papillary carcinoma, 2%
7. Tubular carcinoma, 2%
8. Invasive lobular carcinoma, 10%
9. Rare cancers (adenoid cystic, squamous cell, apocrine)

POST‐GRADUATE GENERAL SURGERY OSCE (By Ali M J) 8


2 BREAST OSCE ‐ Stasions

Prognostic and Predictive Factors for Invasive Breast Cancer:

Traditional Prognostic and Predictive Factors for Invasive Breast Cancer


Tumor Factors Host Factors
Nodal status Age
Tumor size Menopausal status
Histologic/nuclear grade Family history
Lymphatic/vascular invasion Previous breast cancer
Pathologic stage Immunosuppression
Hormone receptor status Nutrition
DNA content (ploidy, S-phase fraction) Prior chemotherapy
Extent of intraductal component Prior radiation therapy
HER-2/neu expression

Male Breast Cancer

DCIS makes up <15% of male breast cancer,


whereas infiltrating ductal carcinoma makes up >85%.

Types of treatment for patients with Breast Cancer:-

Surgery:
Breast Conservative Surgery (BCT):
This is aimed at removing the tumour plus a rim of at least 1 cm of normal breast tissue. This is
commonly referred to as a wide local excision:

Is alternatively called segmental mastectomy, lumpectomy, partial mastectomy, wide local


excision, and tylectomy.

For Stage 0, I, or II invasive breast cancer (less than 3-4 cm), breast-conserving therapy (BCT) is
preferable to total mastectomy because BCT produces survival rates equivalent to those after total
mastectomy

Women with DCIS require only resection of the primary cancer and adjuvant radiation therapy
without assessment of regional lymph nodes

POST‐GRADUATE GENERAL SURGERY OSCE (By Ali M J) 9


2 BREAST OSCE ‐ Stasions

INDICATIONS OF MASTECTOMY
Therapeutic:
1. One or more tumors in separate areas of the breast.
2. Widespread DCIS.
3. Sub-areolar tumor.
4. Large size tumor relatively to breast size (small breast).
5. Risk tumor of further disease (BRCA 1/2 +ve).
6. Previous radiation.
7. Risk from radiation (pregnancy).

Prophylactic mastectomy:
1. Strong family history of breast ca.
2. To obtain optimal symmetry to contralateral breast.
3. For peace of mind following mastectomy for carcinoma of contralateral breast.

Indications of post mastectomy breast radiation:


1. Tumor size > 5 cm.
2. Axillary LN > 4
3. Aggressive histology (diffused vascular invasion) & extra nodal extension.
4. Positive surgical margin.
5. Inflammatory breast cancer
6. Involvement of chest wall.

Nottingham prognostic index = (0.2 × size) + grade + node.


The size in cm.
Grade (1-3 score).
Node score 1-3 (score 1 no node involvement, 2 mean 1-3 nodes & score 3 means 4 or more node
involvement).

10 yr survival
10 yr survival
NPI Prognostic group (surgery & adjuvant
(surgery alone) %
therapy)
< 2.4 excellent 95 95
2.4-3.4 good 85 90
3.41-4.4 Moderate 1 70 79
4.41-5.4 Moderate 2 50 71
> 5.4 poor 20 41

POST‐GRADUATE GENERAL SURGERY OSCE (By Ali M J) 10


2 BREAST OSCE ‐ Stasions

1. Lumpectomy: Surgery to remove a tumor (lump) and a small amount of normal tissue around it.

2. A quadrantectomy: which is more often used outside of North America, refers to removal of the
tumor and about one-fourth of the breast tissue on that side.

3. Partial mastectomy (segmental mastectomy): Surgery to remove the part of the breast that has
cancer and some normal tissue around it. The lining over the chest muscles below the cancer may
also be removed.

4. Subcutaneous: all breast tissue removed but sparing overlying shin, nipple and areola.

5. Skin-sparing mastectomy: all breast tissue and nipple/areola complex, the skin around the
biopsy site, and the skin within 1 to 2 cm of the tumor margin. It usually sacrifices only 5% to
10% of the breast skin.

Include:
Those with multicentric disease.
Invasive carcinoma associated with an extensive intraductal component.
T2 tumors with a difficult-to-interpret mammograms.
Central tumors that would require removal of the nipple/areola complex.

6. Nipple sparing mastectomy (NSM) and areola-sparing mastectomy (ASM): As with the skin-
sparing mastectomy, these are done in conjunction with immediate reconstruction with a plastic
surgeon.

7. Total mastectomy (simple mastectomy): remove the whole breast that has cancer. Some of the
lymph nodes under the arm may be removed for biopsy at the same time as the breast surgery or
after. This is done through a separate incision.

8. Modified radical mastectomy (Patey surgery): Surgery to remove the whole breast that has
cancer, many of the lymph nodes under the arm, the lining over the chest muscles, and
sometimes, part of the chest wall muscles.

9. Radical mastectomy (Halsted): removal of the breast, muscles (including pectoralis major and
pectoralis minor), and axillary lymph nodes.

10. Extended radical mastectomy - as for radical proceudure but also removing internal mammary nodes
(between 2-4th anterior intercostal spaces).

POST‐GRADUATE GENERAL SURGERY OSCE (By Ali M J) 11


2 BREAST OSCE ‐ Stasions

Nonsurgical Breast Cancer Therapies


1) Radiation Therapy
2) Chemotherapy
Adjuvant Chemotherapy
Neoadjuvant (Preoperative) Chemotherapy
3) Antiestrogen Therapy (tamoxifen for premenopausal women or an aromatase inhibitor for
postmenopausal women).
4) Anti–Her-2/Neu Antibody Therapy (trastuzumab/ Herciptin)
5) Ablative Endocrine Therapy (oophorectomy, adrenalectomy, and/or hypophysectomy.

Staging of breast cancer: (ABSIT & pretest)


1. Stage I: T1N0M0
2. Stage II: T (1-2) N1M0
3. Stage III: T (1-3) N (any) M0 or T (any) N2-3 M0
Stage III A: internal mammary LN
Stage III B: involvement of chest wall or skin (including inflammatory breast cancer)
Stage III C: N3
4. Stage IV: T (any) N (any) M1

COLUMBIA CLINICAL CLASSIFICATION SYSTEM

No skin edema, ulceration, or solid fixation of the tumor to the chest wall. Axillary
Stage A
nodes are not involved clinically
No skin edema, ulceration, or solid fixation of the tumor to the chest wall. Clinically
Stage B involved nodes, but less than 2.5 cm in transverse diameter and not fixed to overlying
skin or deeper structures of the axilla
Any one of the five grave signs of advanced breast carcinoma:
(1) Edema of the skin of limited extent (involving less than one-third of the skin over
the breast)
(2) Skin ulceration
Stage C
(3) Solid fixation of the tumor to the chest wall
(4) Massive involvement of axillary lymph nodes (measuring 2.5 cm or more in
transverse diameter)
(5) Fixation of the axillary nodes to overlying skin or deeper structures of the axilla
All other patients with more advanced breast carcinoma, including:
(1) A combination of any two or more of the five grave signs listed under stage C
(2) Extensive edema of the skin (involving more than one-third of the skin over the
breast)
(3) Satellite skin nodules
Stage D
(4) The inflammatory type of carcinoma
(5) Clinically involved supraclavicular lymph nodes
(6) Internal mammary metastases as evidenced by a parasternal tumor
(7) Edema of the arm
(8) Distant metastases

POST‐GRADUATE GENERAL SURGERY OSCE (By Ali M J) 12


2 BREAST OSCE ‐ Stasions

MANCHESTER STAGING SYSTEM:

STAGE CLINICAL FINDING


A NO Mass or 2cm size, no LN involvement.
I
B NO Mass or 2cm size, with micro metastases (0.2-2 mm).
2cm size + LN involvement (Palpable ipsilateral axillary)
A
2-5 cm without LN involvement.
II
2-5 cm + Palpable ipsilateral axillary.
B
˃ 5cm without LN.
A Without mass or any size + Fixed ipsilateral axillary LN.
Involvement of skin
B Chest wall.
III
Inflammatory breast CA.
Internal mammary LN.
C
Supra or infraclavicular LN.
IV Distant Metastasis.

TNM Stage Definitions

Primary tumor (T)


TX: Primary tumor cannot be assessed
T0: No evidence of primary tumor
Tis (DCIS): Ductal carcinoma in situ
Tis: Carcinoma in situ Tis (LCIS): Lobular carcinoma in situ
Tis (Paget's): Paget's disease of the nipple with no tumor.
Tumor 2.0 cm:
T1a: Tumor more than 0.1 but not more than 0.5 cm in greatest dimension
T1:
T1b: Tumor more than 0.5 cm but not more than 1.0 cm in greatest dimension
T1c:Tumor more than 1.0 cm but not more than 2.0 cm in greatest dimension
T2: Tumor more than 2.0 - 5.0 cm.
T3: Tumor ˃ 5.0 cm in greatest dimension
Tumor of any size with direct extension to (a) chest wall or (b) skin, only as described
below:
T4a: Extension to chest wall, not including pectoralis muscle.
T4: T4b: Edema (including peau d'orange) or ulceration of the skin of the breast or
satellite skin nodules confined to thesame breast.
T4c: Both T4a and T4b.
T4d: Inflammatory carcinoma.

POST‐GRADUATE GENERAL SURGERY OSCE (By Ali M J) 13


2 BREAST OSCE ‐ Stasions

Regional lymph nodes (N)


NX: Regional lymph nodes cannot be assessed (e.g., previously removed)
N0: No regional lymph node metastasis
N1: Metastasis to movable ipsilateral axillary lymph node(s)
Metastasis in ipsilateral axillary lymph node(s) fixed or matted, or
N2: clinically apparent ipsilateral internal mammarynodes in the absence of clinically
evident axillary lymph node metastasis
Metastasis in ipsilateral infraclavicular lymph node(s) with or without axillary lymph
node involvement, or in clinically apparent ipsilateral internal mammary lymph
N3: node(s) and in the presence of clinically evident axillary lymph node metastasis; or
metastasis in ipsilateral supraclavicular lymph node(s) with or without internal
axillary or mammary lymph node involvement
Pathologic classification (pN)
Regional lymph nodes cannot be assessed (previously removed, or not removed for
pNX:
pathologic study)
pN0: No regional lymph node metastasis
Metastasis in 1 to 3 axillary lymph nodes, and/or in internal mammary nodes with
pN1: microscopic disease detected by sentinel lymph node dissection but not clinically
apparent
Metastasis in 4 to 9 axillary lymph nodes, or in clinically apparent internal mammary
pN2:
lymph nodes in the absence of axillary lymph node metastasis
Metastasis in 10 or more axillary lymph nodes, or in infraclavicular lymph nodes, or
in clinically apparent ipsilateral internal mammary lymph nodes or in the presence of
pN3: 1 or more positive axillary lymph nodes with clinically negative microscopic
metastasis in internal mammary lymph nodes; or in ipsilateral supraclavicular lymph
nodes
Distant metastasis (M)
MX: Presence of distant metastasis cannot be assessed
M0: No distant metastasis
M1: Distant metastasis present

POST‐GRADUATE GENERAL SURGERY OSCE (By Ali M J) 14


2 BREAST OSCE ‐ Stasions

AJCC stage groupings


Stage 0 Tis, N0, M0
Stage I T1, N0, M0
T0, N1, M0
Stage IIA T1,N1, M0
T2, N0, M0
T2, N1, M0
Stage IIB
T3, N0, M0
T0, N2, M0
T1, N2, M0
Stage IIIA T2, N2, M0
T3, N1, M0
T3, N2, M0

T4, Any N, M0
Stage IIIB
Any T, N3, M0
Stage IV Any T, Any N, M1

POST‐GRADUATE GENERAL SURGERY OSCE (By Ali M J) 15


3 ENDOCRINE OSCE ‐ Stasions

History taken (Cushing’s Syndrome/Disease)

From Above (Head) to (toes)

A. Head:
Thinning of scalp hair (Baldness).
Increase the facial hair (Hirsutism).
Greasy or dry skin with Acne.
Thinning of skin (cracks, splits & bruises easily) (Telangiectasia) Visible vessels on the
cheeks.
Excessive sweating.
Moon face (rounded facial appearance).
Mood change (irritability, Euphoria or depression) (Sleep disturbances/ Insomnia) (Memory
and attention dysfunction).
Visual problem (bitemporal hemianopia) (pituitary tumor pressure on optic chiasm)
Changes in voice? (New deepening of the voice is suggestive of an adrenal virilising
tumour).
Buccal pigmentation.

B. Neck and shoulders


Supra-clavicular fat pads
Increased muscle bulk, for example broadening of the shoulders? (Buffalo Hump) fat pad on
back of neck

C. Abdomen:
Rapid weight gain with Central obesity (sparing of limbs).
Stretch marks (red striae around abdomen, hips).
Bruising.

D. Reproductive System:
Menstrual disorders such as amenorrhea in women and decreased fertility in men

E. Legs:
Edema
Ulceration
Bruising

F. Musculoskeletal:
Muscle weakness/ posture (proximal myopathy).
Limbs or Spine pain/fracture (Osteoporosis).

G. Others:
Poor wound healing (Immune suppression).
Diabetes mellitus (recent).
Hypertension (recent).
Hypercholesterolemia.

H. Past Medical, (taking any steroid containing medications?).


I. Past Surgical. ( Renal Transplant)
J. Family and Social History.

POST‐GRADUATE GENERAL SURGERY OSCE (By Ali M J) 1


3 ENDOCRINE OSCE ‐ Stasions

CUSHING'S SYNDROME EXAMINATION

Wash your hands


Introduce yourself
Ask permission to examine patient
Expose patient, preferably in underwear
Reposition patient lying down

A. General Inspection:
The patient as a whole (for the 'gestalt' picture of Cushing's syndrome):
For central obesity.
For stooped posture (due to osteoporotic damage)
Generalised changes over the body, including:
Easy bruising
Striae
Thin skin
Classical central fat distribution

B. Systemic Examination:
1. Hands and arms for...
Bruising
Thin arms
Ask to measure blood pressure (would be raised)
Shoulder abduction power - reduced in proximal myopathy
Rheumatoid arthritis (suggesting an iatrogenic cause of the Cushingoid features)

2. Face for...
Rounded face (moon face)
Visible vessels on the cheeks suggesting thin skin
Greasy skin
Acne
Hirsutism
At this point you may wish to test Visual fields (looking for a bitemporal hemianopia that may be
the result of an ACTH producing pituitary tumour pressing on the optic chiasm)
Buccal pigmentation.

3. Shoulders for...
Supra-clavicular fat pads
Buffalo hump

4. Abdomen for...
Central obesity
Striae
Bruising
Renal transplant scars (patient may be on long term steroids which is causing the Cushing's
syndrome)

5. Legs for...
Skin ulceration
Ask patient to get up from a sitting position without using hands to test for proximal myopathy

POST‐GRADUATE GENERAL SURGERY OSCE (By Ali M J) 2


3 ENDOCRINE OSCE ‐ Stasions

6. Spine for...
Spinal tenderness (may occur with osteoporosis if vertebral fractures)

C. To end my examination, I would like to:


1. Inspect this patient's drug chart for any steroid use.
2. Do bedside tests:
Carry out a urine dipstick and check BM blood sugar at bedside (Diabetes Mellitus).
Check blood pressure if not already done (Hypertension).
3. Consider further investigations, such as a dexamethasone suppression test.

CUSHING'S SYNDROME INVESTIGATION:

1. Check the cortisol level


24-hour Urinary free cortisol level (1st line)
- Urine 17-Ketosteroid excretion.
- Urine 17-Hydroxysteroid excretion.
Night-time Salivary cortisol testing
Serum Cortisol

If urinary excretion. UFC is elevated (> 4 times the upper limit of normal) between 100 and 150
µg/24h (276 and 414 nmol/24h), then a dexamethasone suppression test may be done to determine
the cause

2. Low dose Dexamethasone Suppression Test


Cortisol & ACTH should decrease due to negative feedback in normal individuals
Dexamethasone 1-2 mg at 11pm, Plasma Cortisol in following 8 AM (in normal people plasma
cortisol decreased to 1.8 µg/mL ( 50 nmol/L))
If still High Plasma ACTH
ACTH decreased [ACTH-independent (adrenal tumor)] It’s Cushing’s Syndrome
Abdominal CT (Adrenal).
ACTH is high High dose Suppression test

3. High dose Dexamethasone Suppression Test (8 mg):


If ACTH decreased ACTH-dependent (pituitary tumor) Head CT or MRI.
If ACTH still elevated mostly ectopic ACTH (small cell CA)

POST‐GRADUATE GENERAL SURGERY OSCE (By Ali M J) 3


4 ABDOMEN OSCE ‐ Stasions

CHANGE OF BOWEL HABIT- BLEEDING PR (HISTORY)

Greet & Introduce yourself. Ask his/her permission

1 Duration
2 Onset (sudden, gradual)
Bleed or not (if yes or already came with bleeding)
Nature (fresh blood, Melena, clots)
3 [bright; fissure or hemorrhoids] [dark; carcinoma, IBD, angiodysplasia, diverticular
disease]
Relation to defecation (before, after or mixed)
change of bowel habits, nature & consistency (diarrhoea; > 3times watery,
constipation or mix of them)
4
[diarrhoea; carcinoma, IBD, diverticular disease, food poisning]
[Constipation; fissure, hemorrhoids, carcinoma, crohn’s stricture]
5 Presence of mucus [rectal carcinoma, IBD]
Abdominal Pain/ Tenesmus during defecation [if yes SIROD CASP3]or
6
abdominal distension
7 Anal pain [anal fissure] or feeling of swelling or tag [hemorrhoid, anal fissure]
8 Sense of incomplete emptying
9 GI symptoms (nausea, vomiting, loss of appetite)
10 Swallowing difficulty, heartburn, indigestion
11 Weight loss [Carcinoma, IBD]
12 Associated symptoms (Jaundice, fever)
13 History of recent foreign travel [infective cause]
14 Anyone in the home have the same problem [food poisoning]
Drug allergy and anti-coagulant (aspirin, warfarin)
Past medical and past surgical (chronic disease, previous operation, radiotherapy,
15
chemotherapy, previous hospital admission, investigation or medication for this
problem)
Past social (Smoking, alcohol) if yes, amount and duration
16
Similar family condition
Thanking

POST‐GRADUATE GENERAL SURGERY OSCE (By Ali M J) 1


4 ABDOMEN OSCE ‐ Stasions

ABDOMINAL (EXAMINATION)

Great and Introduce yourself to the patient, and ask his/her permission.
Positioning and exposure.
Wash/Gel your hands.
Ask (Is it Painful or not? Any other pain region?).

INSPECTION
General inspection: JACCOL + Cachexia
1 From the end of bed see abdominal symmetry, movement with breathing & type of breathing
From right side of patient
contour (scaphoid or distended)
any scar
any dilated veins
2 umbilicus ( position, inverted or everted or flat)
pigmentation
pulsation
cough impulse (hernial orifice & scar)
stomas, drains & catheters
Palpation
1 Asking the patient if there is any painful area and starting away from it
2 Superficial palpation (tenderness , guarding or rigidity and masses
3 Deep palpation of 9 areas for masses
4 Examine liver, spleen & kidney
5 Cough impulse at hernial orifices & aortic aneurysm (expansile pulse & epigastric mass)
Percussion
1 Percussion tenderness (peritonitis)
2 Ascites shifting dullness & transmitted thrill
3 Organ span (liver & spleen)
Auscultation
1 Bowel sound
2 aorta, renal & femoral arteries
3 Over liver for bruit
Extra tests
1 Groin
2 gentalia
3 DRE
4 The back
5 Left supra clavicular lymph nodes
6 Ankle oedema
Finishing Thanking, covering the patient & washing hands

POST‐GRADUATE GENERAL SURGERY OSCE (By Ali M J) 1


4 ABDOMEN OSCE ‐ Stasions

ABDOMINAL MASS (EXAMINATION)

Start by same abdominal inspection then when you find lump:

site
margins
shape
Can get above / bellow it
surface
consistency
temperature
tenderness
Pinch the skin over it
Head raising test (to establish the depth of the mass)
reducible
compressible
Thrill or pulsation
Percussion
Percussion over the mass
Auscultation
Over the mass

On examination of liver, spleen or kidney: look for SPRUTE:


S; Site of enlargement from costal margin & surface
P; Percussion notes
R; Respiratory movement U; Unable to get above it T; Tenderness
E; Edge; smooth or irregular
**On liver examination, arterial bruit may indicate alcoholic hepatitis or cirrhosis. Venous hum is
associated with portal hypertension.

Differential diagnosis of splenohepatomegaly: CHIASMA


C: Congestive (like HF & portal hypertension)
H: Hematological (like myeloproliferative & lymphoproliferative disorders)
I: infection
A: Amyloidosis
S: Storage disorders like Wilson disease, haemochromatosis, Sarcoidosis
M: masses (primary & secondary)
A: Autoimmune & Alcohol cirrhosis

Causes of both spleen & liver enlargements:


1. Portal hypertension
2. Myeloproliferative
3. Lymphoproliferative

POST‐GRADUATE GENERAL SURGERY OSCE (By Ali M J) 2


4 ABDOMEN OSCE ‐ Stasions

How could we differentiate between spleen & kidney enlargement:


1. Spleen descend to RIF & kidney to LIF
2. Kidney is ballotable, spleen not
3. It might be possible to get above kidney & spleen not
4. Spleen has palpable notch & kidney not
5. The spleen is usually dull & the kidney may be resonant due to overlying bowel
6. The spleen may have a friction rub; the kidney not.

POST‐GRADUATE GENERAL SURGERY OSCE (By Ali M J) 3


4 ABDOMEN OSCE ‐ Stasions

DYSPHAGIA (HISTORY)

Greet & Introduce yourself. Ask his/her permission

1 Onset of Dysphagia (how long)


2 Dysphagia to solids or to liquids (or both)
3 Progression of dysphagia (sudden or gradual; in general) & (from liquids to solids)
4 Constant (pharyngeal, laryngeal Ca) or intermittent
5 Painful (odynophagia) or painless Dysphagia
6 Site of sticking of food (back of throat, neck, chest)
7 Any food, or drink aggravate the condition (Hot; GERD)
8 Rregurgitation (do you have)
9 Ask about positions that aggravate regurgitation
10 Nature, smell & taste of regurgitated material
11 Aspiration & chalking spills (cough)
12 History of recurrent respiratory infections, or change in breath (strider)
13 Associated neck mass or lump (Lymph node, pharyngeal pouch)
14 Changing in voice (hoarseness; pharyngeal Ca)
Pain elsewhere & other associated symptoms (epigastric pain and heart burn; GERD,
15
otalgia; pharyngeal Ca)
16 Weight loss
17 Haematemesis or melena
18 Bowel motion/diarrhea or constipation
Medical illnesses, Drugs allergy, previous Hospitalization previous Surgery,
19 Radiation, Chemotherapy, previous Investigation endoscopy or Treatment for this
problem
20 Family history and social history (smoking, drinking, family similar condition)

POST‐GRADUATE GENERAL SURGERY OSCE (By Ali M J) 1


4 ABDOMEN OSCE ‐ Stasions

JAUNDICE (HISTORY)

Greet & Introduce yourself. Ask his/her permission

1 Duration [long standing; alcohol cirrhosis] [short duration; CBD stone]


2 Previous attack
3 Change in urine colour
4 Change in stool colour
5 Feel itchy (bile salt), scratching
6 Bruising tendency (due to hepatocellular damage)
Pain or painless:
7 Previous biliary colic (RUQ pain associated with fatty meal or Constant epigastric with
weight loss) (Site, Intensity, Radiation, Onset ….etc.)
8 Fever
9 Nausea, vomiting
10 Hematemesis, malena
11 Hematuria
12 Loss of appetite
13 Weight loss [in hepatitis; anorexia & Malignancy]
14 Change in bowel habit (constipation, diarrhea)
15 Joint pain & Back pain [Hepatitis]
16 Sore throat (Epstien –Barr virus)
17 Foreign travel (hepatitis A)
18 Blood transfusion (Hepatitis B, C), IV drugs, Tattoos
19 Sexual relations
Drugs:
(contraceptive pills, diltiazem, diclofenac…. Hepatic Jaund.) (phenothiazines Heparin,
20 Augmentin, Erythromycin…… obstrtructive Jaund.).
drugs allergy
Previous episode [what was the cause, the investigations & how was treated]
Past social (Smoking, alcohol) if yes, amount and duration
21
Similar family condition
Thanking the patient

POST‐GRADUATE GENERAL SURGERY OSCE (By Ali M J) 1


4 ABDOMEN OSCE ‐ Stasions

JUINDCE PREPARATION

POST‐GRADUATE GENERAL SURGERY OSCE (By Ali M J) 2


4 ABDOMEN OSCE ‐ Stasions

ABDOMINAL PAIN (HISTORY)

Greet & Introduce yourself. Ask his/her permission

Occupation
Site (where is)
For how long (onset)
sudden or gradual
Duration (how long does the pain last)
Intermittent or constant
Intensity (from 1-10), activity limitation, wake him at night
Character (sharp, heavy, dull, burning or colicky)
Radiation
Aggravating factors (work, food directly after food or few hrs later, fatty meal)
relieving factors
Similar previous attacks of pain, is it periodic (more in certain time or weather)
ASSOCIATED SYMPTOMS
Nausea & Vomitting, heart burn
Hematemesis
Changes in bowel habit
Blood in the stool (malena)
Loss of appetite
Weight loss
sweeting, fever,
Jaundice
palpitation, SOB, cough,
loss of conscious
Urinary symptoms
Vaginal discharge (if the patient is female)
Last period and Possibility of being pregnant
Past medical:
Chronic disease (DM, HT, HF, liver diseases).
Drugs (NSAIDs), or medication for this problem
Allergies.
Previous hospital admission, investigation (especially ENDOSCOPY).
Past surgical:
Previous operation.
Radiotherapy.
Chemotherapy.
Family Social History:
Similar family condition, Family history of malignancies.
Smoking & Alcohol.
THANKING the patient.

POST‐GRADUATE GENERAL SURGERY OSCE (By Ali M J) 1


4 ABDOMEN OSCE ‐ Stasions

VOMITING / HEMATEMESIS (UGI bleeding) (HISTORY)

Greet & Introduce yourself. Ask his/her permission

Present Illness:
1 Duration
2 Quantity
3 Appearance (blood + bile)
4 Any clots
5 Any abdominal pain
6 Bowel symptoms
7 Stool colour
8 Any syncope/ any chest pain/ any sweating
9 Need for blood transfusion?
10 Any weight loss
11 Jaundice
12 Fever
Past Medical History:
13 Peptic ulcer
14 Liver disease (Abdominal distension, dilated veins, hand palms erythema)
15 Any cancers
16 Prior bleeding
17 Cad
18 Previous surgeries
19 Medications: use of NSAIDS/Aspirin
20 Family History
21 Social History (particularly alcohol use)
THANKs the patient

POST‐GRADUATE GENERAL SURGERY OSCE (By Ali M J) 1


4 ABDOMEN OSCE ‐ Stasions

VOMITING / HEMATEMESIS (UGI bleeding) (EXAMINATION)

inspection
Assess level of consciousness
Look for signs of chronic liver disease (palmer erythema, clubbing, spider
angioma, gynecomastia, jaundice, testicular atrophy, ascitis,
hepatosplenomegaly)
palpation
Vital signs
Examine abdomen: inspection, palpation (light/deep), percussion,
auscultation
Ensure that the patient is properly draped
Intend to perform DRE
Finishing

POST‐GRADUATE GENERAL SURGERY OSCE (By Ali M J) 2


4 ABDOMEN OSCE ‐ Stasions

WEIGHT LOSS (HISTORY)

Greet & Introduce yourself. Ask his/her permission

1 Duration
2 Loss of weight in Kg
3 Loss of appetite, weakness & easy fatigability
GI symptoms
4 dysphagia
5 vomiting
6 diarrhea & constipation
7 change of bowel habits
BLEEDING
8 GIT Hematemesis, melena
9 URINARY Hematuria
10 RESPIRATORY hemoptysis
11 GYNECOLOGY vaginal bleeding
THYROTOXICOSIS SYMPTOMS
12 Intolerance to heat
13 sweating
14 Tachycardia, palpitation
15 nervousness
16 PSYCHOLOGICAL STATUS
17 Past medical history (DM, chronic illness)
18 Past surgical history (previous surgery, Tumor, chemotherapy, radiotherapy)
19 Family Hx and Smoking, Alcohol Hx
THANKS the patient

POST‐GRADUATE GENERAL SURGERY OSCE (By Ali M J) 1


4 ABDOMEN OSCE ‐ Stasions

STOMA (HISTORY)

Greet & Introduce yourself. Ask his/her permission

1 Duration
2 Elective or emergency
3 Cause of surgery (permanent or temporary)
4 Site
5 Colostomy bag (content, blood, amount)
6 Change of colostomy bag
7 Abdominal pain
8 nausea, vomiting
9 bleeding
10 diarrhea
11 prolapsed
12 Skin condition
13 wound infection
14 Fever
15 weight loss
Past medical
16 past surgical (chronic disease, previous operation, radiotherapy, chemotherapy,
previous hospital admission, investigation or medication for this problem)
17 Family hx
18 Smoking, alcohol
THANKS the patient

POST‐GRADUATE GENERAL SURGERY OSCE (By Ali M J) 1


4 ABDOMEN OSCE ‐ Stasions

STOMA (EXAMINATION)

Great and Introduce yourself to the patient, and ask his/her permission.
Positioning and exposure.
Wash/Gel your hands.
Ask (Is it Painful or not? Any other pain region?).

INSPECTION
ABDOMEN (distended, scar, erythema….etc)
Scar (medical prescription)
No midline scar (defunctioning, or laparoscopy)
Ask the patient to cough or raise his legs (parastomal hernia)
Site (RIF, LIF, epigastric, transverse…etc.)
Covering (with bag or had been removed) & comment on the bag content [urine,
formed stool, liquid stool]
Caliber shape: spout (ileostomy) or flush colostomy
Lumen number [spout, end; one opening, loop; with a common opening or 2 separate
openings (double-barrelled)]
Maybe a colostomy with a urinary diversion [pelvic clearance]
State of stoma (Healthy well-constructed, ischemic, retracted, prolapsed) any mucosal
inflammation or polyps
Surrounding skins (excoriations)
Palpation (Digital stomal examination) begin with distal opening
Any stenosis
Confirming weather single or double lumen
I would like to complete by examining:
The rest of the abdomen.
Perineum (patent anal canal or closed perineum).
Asses stoma position during sitting, lying & standing
Assess the patient's fluid status (hydration; high output stmoa).
THANKING and COVER the patient.

POST‐GRADUATE GENERAL SURGERY OSCE (By Ali M J) 2


4 ABDOMEN OSCE ‐ Stasions

Indications

1. Feeding (gastrostomy, jejunostomy)


2. Lavage (appendicostomy) (caecostomy; on table lavage)
3. Decompression (bypass)
4. Diversion (anastomosis, urinary)
5. Exteriorization (hartmann’s; perforation or ischemic obsstruction) (Permanent stoma,
panproctocolectomy)

Complications
Specific/ technical:
1. Ischemic/gangrene.
2. Obstruction.
3. Haemorrhage.
4. Retraction.
5. Prolapse/intussusception.
6. Stenosis.
7. Parastomal hernia.
8. Skin excoriation.
9. Odour & flatus.

General:
1. Diarrhoea (water and electrolyte imbalance, especially hypokalemia)
2. Nutritional disorders (VB12 deficiency; megaloblastic anemia, microcytic
normochromic anemia).
3. Stones (gallstones, renal stone) loss of terminal ileum and bile salt absorption, also
excessive water loss.
4. Residual disease (crohn’s; parastomal fistula).
5. short gut syndrome: fluid/electrolyte loss
6. Psychosexual.

Prepare patient for stoma formation:


1. Physical and psychosocial.
2. Explain the indication and complications.
3. Mark the site (with stoma nurse).
Standing up (normally can see the stoma, and easy accessible by the patient).
Within rectus abdominus muscle.
Away from scar, skin crease or umbilicus.
Away from bony points or waistline of clothes.

POST‐GRADUATE GENERAL SURGERY OSCE (By Ali M J) 3


5 POST‐OPERATIVE OSCE ‐ Stasions

POST-LAPAROTOMY (EXAMINATION)

Great and Introduce yourself to the patient, and ask his/her permission.
Positioning and exposure.
Wash/Gel your hands.
Ask (Is it Painful or not? Any other pain region?).

General appearance (The patient looks ill or fine), position


Vital signs
1 Pulse rate
2 Blood pressure
3 Respiratory rate
4 Temperature
Examination
1 Hydration state
2 Checking for color changes ( pallor , jaundice , cyanosis )
3 Auscultation of the chest anteriorly
4 Auscultation of the chest posteriorly (lung bases)
5 Inspection of the abdomen
6 Examine site of operation
7 Auscultation for bowel sound
8 Examination of the lower limbs for DVT & pitting oedema
Tube checking
1 Intravenous access examination (cannula site like thrombophlebitis, type of fluids..)
2 NG tube
3 drains
4 Urine output (Folys catheter or asking the patient )
Finishing
1 Thanking and covering the patient
2 Washing hands

POST‐GRADUATE GENERAL SURGERY OSCE (By Ali M J) 1


5 POST‐OPERATIVE OSCE ‐ Stasions

POST-THYROIDECTOMY (EXAMINATION)

Great and Introduce yourself to the patient, and ask his/her permission.
Positioning and exposure.
Wash/Gel your hands.
Ask (Is it Painful or not? Any other pain region?).

General Examination
General appearance ( conscious or not , well or tired , pallor , jaundice , cyanosis
1
and respiratory state)
2 Pulse examination (rate,rhythm,volume)
3 Blood pressure and trousses sign
4 Hydration state
5 Checking for pallor
6 Checking for jaundice
7 Auscultation of the chest anteriorly
8 Auscultation of the chest posteriorly (lung bases)
Specific examination
9 Removal of dressing and wound inspection ( checking for any heamatoma
10 Patient voice
11 Chvosticks sign
12 Ask the patient to cough
Tube checking
13 Intravenous access examination
14 Drains
Thanking and covering the patient

POST‐GRADUATE GENERAL SURGERY OSCE (By Ali M J) 2


5 POST‐OPERATIVE OSCE ‐ Stasions

POSTOPERATIVE FEVER (VIVA)

What are the causes of post-operative fever? Give the likely post-operative days related to the
cause.

6W
(Wonder drug, Wings (veins), Wind, Wound, Walking, Water)
Same time: (Wonder drug)
1 Transfusion reactions
2 Drug reactions
Day 1:
3 Inflammatory response to surgical trauma
4 Thrombophlebitis (at any day) (Weins/Wings)
Day 2: (Wind)
5 Atelectasis of the lung
Day 3:
6 Systemic bacteremia/fungemia/viremia
Day 5: (Wound)
7 Superficial and deep wound infection
After day 5:
8 Intra-abdominal abscess
Day 7 & on:
Deep vein thrombosis (DVT) (Walking)
Urinary tract infection (Water)
9 Leaking anastomosis
Clostridium difficile diarrhea
Deep wound infection
Abscess
Others
Infected prosthetic material
10 Collagen/Vascular disease
Occult bacteremia
Neoplasm

POST‐GRADUATE GENERAL SURGERY OSCE (By Ali M J) 3


5 POST‐OPERATIVE OSCE ‐ Stasions

CAUSES OF POSTOPERATIVE JAUNDICE?

A) Jaundice that develop within 48 hours of the operation:


1. Breakdown of RBCs due to underlying hemolytic anemia or precipitated by the
administration of specific drugs (e.g., sulfa drugs in a patient who has G6PD).
2. Multiple blood transfusions (particularly with stored blood).
3. Transfusion reaction.
4. The resorption of a large hematoma.
5. Cardiopulmonary bypass or the insertion of a prosthetic valve.
6. Gilbert syndrome may first manifest itself early in the postoperative period.
7. Dubin-Johnson syndrome, which is an inherited disorder of bilirubin metabolism.
This condition is usually self-limited and is characterized by the presence of a melanin
like pigment in the liver.
B) Within 5 to 10 days after operation
1. Intraoperative hypotension or hypoxemia
2. the early development of heart failure
3. other end-organ damage (e.g., acute tubular necrosis) due to impairment of renal
function(mild hyper bilirubinemia).

(Within 7-10 days)


4. medication-induced hepatitis attributable to an anesthetic agent.
5. administration of antibiotics or other medications used in the perioperative setting

C) After the first week:

1. Intrahepatic cholestasis is often a manifestation of a septic response and usually


presents in the setting of overt infection, particularly in patients MOFS. Gram-
negative sepsis from an intra-abdominal source is typical.
2. Patients receiving TPN. It may be attributable to steatosis, particularly with formulas
containing large amounts of carbohydrates. In addition, decreased export of bilirubin
from the hepatocytes may lead to cholestasis.
3. Acalculous cholecystitis or even ductal obstruction may develop as a result of sludge
in the gallbladder and the CBD.
4. Unsuspected hepatic or post-hepatic causes (e.g., occult cirrhosis,
choledocholithiasis, or cholecystitis).
5. Development of thyrotoxicosis (a rare cause).
6. Benign postoperative cholestasis, a primarily cholestatic, self-limited process with
no clearly demonstrable cause that typically arises within 2 to 10 days after operation.
D) In the late postoperative period (within 5 to 12 weeks of operation)
the development of non-A, non-B, non-C viral hepatitis after transfusion of blood products

POST‐GRADUATE GENERAL SURGERY OSCE (By Ali M J) 4


5 POST‐OPERATIVE OSCE ‐ Stasions

POST‐GRADUATE GENERAL SURGERY OSCE (By Ali M J) 5


6 INGUINAL OSCE ‐ Stasions

GROIN LUMP (history)

1 Duration
2 Onset (gradual) (sudden; is it related with lifting)
3 Occupation or leisure involves a lot of lifting?
4 Increase in size (with muscular efforts, cough, straining)
5 Reducible or not? When lie down or by yourself [hernia, saphena varix]
6 Previous lumps or swelling similar [recurrent hernia]
7 Painful [incarcerated hernia, abscess]
8 Discharge [abscess, infected LN]
9 Fever, night sweeting, weight loss [lymphoma, TB]
GI symptoms; abdominal pain, Fullness, bloating or dyspeptic symptoms. Nausea or
10
vomiting
11 Chronic constipation
12 Chronic cough
13 Urinary symptoms
Past medical; diabetes, jaundice, tuberculosis, ascitis, hypertension. Drug history;
14 steroids, cytotoxic drugs
previous hospital admission, investigation or medication for this problem
15 Social history; smoking, alcohol
THANKING the patient

Differential diagnosis of lump in groin: "L-SHAPE"


Lymph nodes & Lipoma of cord
Sapheno-varix, Skin & Subcutaneous lesions
Hernia (inguinal & femoral)
Aneurysm of femoral A
Psoas abscess & bursa
Ectopic & undescended testis

POST‐GRADUATE GENERAL SURGERY OSCE (By Ali M J) 1


6 INGUINAL OSCE ‐ Stasions

INGUINAL (EXAMINATION)

Great and Introduce yourself to the patient, and ask his/her permission.
Positioning and exposure.
Wash/Gel your hands.
Ask (Is it Painful or not? Any other pain region?).

Introduction (stand patient up & expose groin, genetalia & abdomen)


Inspection
Define character of the lump in groin Scar (especially on any lump)
Cough 1: cough reflex of affected side Cough 2: = = = contralateral side
Palpation
1 Stand to the patient side with one hand on their back & other on the lump
2 Any pain?
3 Can you get above lump?
4 Lump relation to pubic tubercle (femoral or inguinal)
5 Lump characters
6 Cough 3: expansile cough impulse
Reducibility: ask the patient to do that if possible, (direct course – direct hernia, oblique
7 course
–indirect H.)
Cough 4: put one finger on pubic tubercle & ask the patient to cough (note relation
8
protruded lump to the tubercle, inguinal or femoral)
9 Cough 5: deep ring occlusion test
Auscultation
Bowel sound for viability of bowels
Extra tests
1 Full history
2 Examine of contralateral groin
3 Genetalia examination for coincidental hydrocele or varicocele
4 DRE
5 Regional lymph nodes
6 Abdominal examination
Finishing
1 Thanking, covering the patient & washing hands
2 Summarise the case & give differential diagnosis

POST‐GRADUATE GENERAL SURGERY OSCE (By Ali M J) 2


6 INGUINAL OSCE ‐ Stasions

EXAMINATION OF MALE GENETALIA

Introduction (stand patient up & expose groin, genetalia & abdomen)


Inspection
Define character of the lump in groin
Scar (on any lump & including posterior aspect of scrotum) Cough reflex
Palpation
1 Stand to the patient side with one hand on their back & other on the lump
2 Any pain?
3 Can you get above lump?
4 Lump relation to the testis or scrotal skin & scrotal skin pathology
5 Lump characters
6 Expansile cough impulse
7 Transillumination test
8 Examine the patient in standing position to avoid missing a varicocele
Extra tests
1 Full history
2 Examine of contralateral groin
3 DRE
4 Regional lymph nodes
5 Abdominal examination
Finishing
1 Thanking, covering the patient & washing hands
2 Summarise the case & give differential diagnosis

POST‐GRADUATE GENERAL SURGERY OSCE (By Ali M J) 3


7 DRE OSCE ‐ Stasions

DIGITAL RECTAL EXAMINATION (DRE)

1 Great & introduce yourself to the patient & EXPLAIN the procedure
Take consent (Verbal consent only for DRE, but do document in the notes that you gained
2
consent from the appropriate person)
Ensure privacy with assistance nurse (chaperone) (private room, curtains good enough)
3
Keep as much of the patient covered as possible
Positions:
Left lateral (bring their knees right up to their chest)
4 Knee elbow position (jack-knife)
Modified lithotomy (patient on back, knees flexed)
Leaning forward on bench.
5 Glove both hands (non-sterile gloves)
Gently separate the buttocks for inspection of the perineum:
6 External skin lesion (e.g. warts, candidiasis, herpes simplex.)
prolapsed piles, rectal prolapse (ask patient to strain), abscess, sentinel piles, fissure, fistula
Digital examination;
Ask about tenderness (if yes complete under GA)
7
Generously lubricate the gloved index finger, inform the patient that you are going to insert
your finger.
8
Any lubricant can be used (lidocaine used for slight pain and should wait to 5 minutes) (severe
pain need GA)
Left hand on the patient back (stabilize the patient) right hand index down natal cleft with
slight pressure on the anal verge (to relax puborectalis muscle, before insertion) insert your
9 index finger into the anus.

Ask the patient to squeeze to check sphenictor tone

10 Perform a full 360 ° sweep assessing Sweep the anterior, lateral & posterior walls of the
rectum
you need to turn away from the patient and pronate your wrist
Sweep your finger over the prostate gland (anteriorly through the rectal wall)
11 Identify the two lobes, and the longitudinal groove (median sulcus)
Note the size, nodularity, consistency and tenderness of the prostate (prostatitis)
Finding:
Rectal mucosa: Normal mucosa feels uniformly smooth and
pliable to palpate
12
prostate: (normal) Prominent median sulcus Smooth, rubbery consistency
(Ca) Asymmetric shape, Hard consistency, Discrete nodule may be palpable, Median sulcus
often obscure
Withdrawal your index and inspect the colour of any soiling on your glove (faeces, mucous,
13
blood)
Ending:
Offer the patient a tissue
14 Allow the patient to get dressed, sit down and
prepare themselves for discussing results
Explain your findings to the patient
15
Negotiate a follow up plan / tests /investigations like Proctoscopy, or Sigmoidoscopy
Address the patients concerns

POST‐GRADUATE GENERAL SURGERY OSCE (By Ali M J) 1


7 DRE OSCE ‐ Stasions

Reasons for DRE:

1. Suspected Appendicitis
2. Change in bowel habit
3. PR bleeding
4. As a part of abdominal exam
5. Pelvic or spinal trauma
6. Genitourinary problems

PROCTOHEAL composition:

1. Fluocinolone acetonide corticosteroid (Topical corticosteroids share anti-inflammatory, anti-


pruritic and vasoconstrictive actions)
2. Menthol (antipruritic, analgesic, decongestant)
3. Bismuth subgallate (lubricating properties)
4. Lidocaine hydrochloride (analgesic)

POST‐GRADUATE GENERAL SURGERY OSCE (By Ali M J) 2


8 LIMB OSCE ‐ Stasions

PERIPHERAL VASCULAR DISEASE (HISTORY)


CLUDICATION

Greet & Introduce yourself. Ask his/her permission


Begin by asking: Name, Age, Occupation, if not mentioned previously

6 Ps
Pain:
Onset
Duration
Location
Radiation
Alleviating factors
Provoking factors
Paresthesias
Pallor
Paralysis
Pulseless
Poikilothermic (temperature change, cold)
Past Medical History: (risk factors)
Diabetes mellitus
Coronary artery disease
Hypertension
Dyslipidemia
Family History: CAD, HTN, Stroke, DM
Social History:
Smoking
Alcohol
Cocaine / heroine abuse
Diet
Activity
THANKING the patient.

POST‐GRADUATE GENERAL SURGERY OSCE (By Ali M J) 1


8 LIMB OSCE ‐ Stasions

PAD (EXAMINATION)
Also for DIABETIC FOOT

Great and Introduce yourself to the patient, and ask his/her permission.
Positioning and exposure.
Wash/Gel your hands.
Ask (Is it Painful or not? Any other pain region?).

Inspection
General:
1 Look around bed for aids, O2, GTN
Look for pt as a whole; unwell, on pain, shortness of breath, cyanosis, obesity
2 Nails changes
Skin changes ( Color, Ulcer , Gangrene, digital amputation) if found ULCER:
3
Site, Size, Shape, Edge, Floor, Surrounding skin.
4 Trophic Changes ( Loss of hair , Muscle wasting )
5 Don’t forget examine between toes, pressure areas & bilaterally
Palpation
6 Tenderness
7 Temperature
8 Pitting edema
9 Base of the ulcer ( induration and fixity to deep structures)
10 Capillary Refilling Time
pulses examination(Abdominal Aorta, Femoral, Popliteal, Posterior Tibial & Dorsalis
11
Pedis)
Auscultation (bruit)
12 Aorta, renal & iliac aa
13 Femoral
14 Adductor hiatus
Special examination
15 ABPI
16 Buerger’s test
17 DeWeese test (disappearance of previously palpable pulse after walking exercise)
Nerve functions
18 Temperature sensation
19 Touch sensation
20 Motor
Finishing Thanking the patient , cover pt & washing hands

POST‐GRADUATE GENERAL SURGERY OSCE (By Ali M J) 2


8 LIMB OSCE ‐ Stasions

EXAMINATION OF VARICOSE VEINS


Exposure (from hip, keep genitalia covered)

Inspection
General:
Look around bed for aids & support stocking
Look for pt as a whole; unwell, on pain, shortness of breath, cyanosis, obesity
Ask the pt to stand with one leg in front the other, you must inspect from the front, side &
behind.
Skin (scar, edema, eczema, ulcer, hair loss, hemosiderosis, lipodermatosclerosis) due to
venous HT … especial around med malleolus
Asses varicose vein distribution & location
Asses Sapheno-varix
Site and size include saphena varix (SFJ; inguinal medial to femora A pulse), LSV front
and lateral & behind for SSV (below knee)
Palpation (Ask pt if there is pain)
Tenderness
Temperature
Pitting oedema
Palpate the course of long & short saphenous veins
palpate the groin for regional LN
Palpate for Sapheno-varix
Perform cough test by asking pt to cough while your index finger on the sapheno-femoral
junction
Percussion Test (Chevrier,s Sign)
Tap proximal & palpate distally, to identify venous valvular incompetency
Tap distally & palpate proximally, to identify venous lumen patency
Auscultation (bruit)
Listen for bruit (A-V fistula)
Special test
Tourniquet test: also known as Brodie-Trendelenburg test (if the finger is used instead of
tourniquet over SFJ). Place pt in supine position, elevate leg, empty the leg veins & put
tourniquet on mid thigh. Ask pt to stand, if incompetence is above it, the various V will be
controlled & if it is bellow the V will be refilled. The test can be repeated using multiple
positions.
Perthes test: place the tourniquet on the thigh & ask pt to stand on their toes. If the veins
enlarged or pt experience pain, the deep vein likely to be involved.
Hand held Doppler ultrasound
Further considerations
ABPI
Abdominal examination
Duplex US
Thanking the patient , cover pt

POST‐GRADUATE GENERAL SURGERY OSCE (By Ali M J) 3


8 LIMB OSCE ‐ Stasions

DVT (HISTORY)

Greet & Introduce yourself. Ask his/her permission


Begin by asking: Name, Age, Occupation, if not mentioned previously

1 Onset of the pain


2 Fever
3 Chest pain
4 Shortness of breath
5 Cough, hemoptysis
Risk Factors
6 Age
7 Occupation
8 Hx of recent Surgery Type of Surgery (pelvic, major…)
9 Duration of surgery
10 Body Weight (obesity)
11 Hx of recent immobilization (travelling ˃ 6h)
12 Varicose vein
13 Trauma
14 Pregnant
15 OCP
16 Blood problem (clotting abnormalities)
17 Past medical previous attack, drugs
18 Past Surgical
19 Family hx of blood clotting
20 Smoking, alcohol
Thanks the patient

DVT exam: Examination lower limb as PVD:

Begin by general look General: for aids, O2, GTN, unwell, on pain, shortness of breath,
cyanosis, and obesity.
Inspection
Palpation (tenderness, temperature, vascular, neurology) + Hoffman sign (calf pain with
dorsiflexion of right foot)
Chest exam (Listen to the lungs in four places)

POST‐GRADUATE GENERAL SURGERY OSCE (By Ali M J) 4


8 LIMB OSCE ‐ Stasions

ARTERIAL EXAMINATION OF THE UPPER LIMB

Greeting the patient , Self introduction , Permission


Hand washing
Asking the patient if there is any pain
Positioning and exposure (upper limb and chest)

Inspection with comparison of the two sides


1 Nail changes ( tar staining , clubbing , brittle nails , )
2 Skin changes (cyanosis , gangrene )
3 Muscle wasting
Palpation with comparison of the two sides
4 Temp
5 Capillary refill
6 Radial pulse
7 Brachial pulse
8 Axillary artery
9 Subclavian artery
10 Neurological assessment (Sensory, motor, Autonomic)
Auscultation
11 Infraclavicular fossa
12 Supraclavicular fossa
Special tests
13 Allens test
Thanks the patient

POST‐GRADUATE GENERAL SURGERY OSCE (By Ali M J) 5


8 LIMB OSCE ‐ Stasions

HEPARIN

Dosing:
1. Give heparin and warfarin jointly for 5-7 days.
2. 5000 units IV one time as a bolus dose followed by 1300 units/hour by continuous IV
infusion. Alternatively, a bolus dose of 80 units/kg IV one time followed by 18
units/kg/hour by continuous IV infusion may be used.
3. Obtain APTT at 4-6 hrs and keep APTT in a range that corresponds to a plasma
heparin level of 0.2-0.4 u/ml.
4. Start warfarin on day one at 5 mg and dose daily with the estimated daily maintenance
dose or start the estimated daily maintenance dose (2-5 mg.)
5. Obtain platelet count every 3-5 days of heparin therapy up to 21 days.
6. Stop heparin thereafter when PT gives an INR of 2.0-3.0.
7. Continue warfarin at an INR of 2.0-3.0.

Complication:
1. Bleeding
2. Heparine induced thrombocytopenia
3. Intra-arterial thrombosis.
4. Osteoporosis.
5. Skin necrosis.

low-molecular-weight heparin (LMWH) Differences from heparin (i.e. "unfractioned


heparin") include:

1. Average molecular weight: heparin is about 15 kDa and LMWH is about 4.5 kDa.
2. Less frequent subcutaneous dosing than for heparin.
3. No need for monitoring of the APTT coagulation parameter as required for high dose
heparin.
4. Possibly a smaller risk of bleeding.
5. Smaller risk of osteoporosis in long-term use.
6. Smaller risk of heparin-induced thrombocytopenia, a potential side effect of heparin.
7. The anticoagulant effects of heparin are typically reversible with protamine sulfate,
while protamine's effect on LMWH is limited.
8. Has less of an effect on thrombin compared to heparin, but about the same effect on
Factor Xa.

POST‐GRADUATE GENERAL SURGERY OSCE (By Ali M J) 6


8 LIMB OSCE ‐ Stasions

ABI 0.9-1.2 Normal


0.5-0.9 Mild (exercise pain).
0.3-.05 Moderate (rest pain).
0.3 Severe (gangrene).

RISK FACTORS:

1. Age: 60-80
2. Male
3. Indo-Asians and African-Americans
4. Familial history of PAD, myocardial infarction, coronary heart disease, or stroke
5. Diabetes
6. HT
7. Hyperlipidemia
8. Obesity
9. Systemic atherosclerosis
10. Myocardial infarction
11. Coronary heart disease
12. Stroke
13. Renal artery stenosis
14. Leriche syndrome
15. Cardiac arrhythmias, including atrial fibrillation, with emboli to the extremities
16. Hypercoagulable states due to elevation of several coagulation factors

Assessment of Severity (FONTAINE SYSTEM):

Stage I: Asymptomatic.
Stage II: Mild claudication pain in limb
IIA: Claudication at a distance of greater than 200 metres.
IIB: Claudication distance of less than 200 metres
Stage III: Rest pain, mostly in the feet
Stage IV: Necrosis and/or gangrene of the limb

A more recent classification by (RUTHERFORD) consists of three grades and six categories:
Category 0: Asymptomatic
Category 1: Mild claudication
Category 2: Moderate claudication
Category 3: Severe claudication
Category 4: Rest pain
Category 5: Minor tissue loss; Ischemic ulceration not exceeding ulcer of the digits of the foot
Category 6: Major tissue loss; Severe ischemic ulcers or frank gangrene

POST‐GRADUATE GENERAL SURGERY OSCE (By Ali M J) 7


8 LIMB OSCE ‐ Stasions

A) Acute limb ischemia

Sudden decrease in limb perfusion that causes a potential threat to limb viability (manifested by
ischemic rest pain, ischemic ulcers, and/or gangrene)
In patients who present within two weeks of the acute event (if >2 weeks, it is considered chronic
ischaemia).

CAUSES:

1. Arterial embolism
Most common cause of acute limb ischaemia (60-80% of the time)
The most likely source of embolus is the heart (80%), of which 70% is due to atrial fibrillation, 20%
to AMI with left ventricular mural thrombus, and a small proportion to prosthetic heart valves

Most common sites where emboli lodge:


Bifurcation of the femoral artery (most common site)
Trifurcation of the popliteal artery (next most common site in the lower limb)
Aortic bifurcation
External and internal iliacs
Arm (about 20% of emboli)

2. Acute thrombosis
Thrombosis of a previously stenotic but patent artery (atherosclerotic vessel)
Other less common causes of acute thrombosis include the arteritides (usually affecting medium-
sized arteries), ergotism, and hypercoagulable states (notably antiphospholipid syndrome).

3. Arterial trauma
4. Dissecting aneurysm

Classification of severity
Viable: No immediate threat of tissue loss
Threatened: Salvageable if re-vascularized promptly
Non-viable: Limb cannot be salvaged and has to be amputated, no emergency to operate

Viable Threatened Non-viable


Pain Mild Severe Variable
Capillary refill Intact Delayed Absent (fixed stain)
Motor deficit None Partial Complete
Sensory deficit None Partial Complete
Arterial Doppler Audible Inaudible Inaudible
Venous Doppler Audible Audible Inaudible
Treatment Urgent work-up Emergency surgery Amputation

POST‐GRADUATE GENERAL SURGERY OSCE (By Ali M J) 8


8 LIMB OSCE ‐ Stasions

B) CHRONIC LIMB ISCHAEMIA

a. Critical:- a potential threat to limb viability (manifested by ischemic rest pain, ischemic
ulcers, and/or gangrene) in patients who present more than two weeks after the acute event
(the converse of the definition of acute limb ischaemia).
b. non-critical limb ischaemia:-
symptomatic (usually claudication)
asymptomatic.

CAUSES of vascular claudication

Most commonly atherosclerotic disease


Other less common causes: ergot toxicity, Takayasu’s arteritis, Buerger’s disease
(thromboangiitis obliterans), vasospasm

Neurogenic claudication
Vascular intermittent claudication needs to be differentiated from neurogenic claudication which can
also present as pain in the lower limb on exertion

The characteristic of neurogenic claudication is “park bench to park bench” where the patient
has to sit down and flex the spine to relieve the pain (pain results from compression of the
cord and spinal nerves in spinal stenosis; extension of the spine further narrows the spinal
canal while flexion widens it)
“Claudication distance” of neurogenic claudication is more variable
Pulses will be absent/diminished in vascular but not in neurogenic claudication

DIAGNOSTIC TESTS AND INTERPRETATION


LAB
CBC and platelets
Electrolytes, BUN, creatinine, glucose
Coagulation studies
Creatine phosphokinase to evaluate for ischemia.
ECG if patient is older than 40 yrs old.
cardiac enzymes (If suspecting an AMI with mural thrombus).
Special tests for suspected etiologies:
Hold blood for hypercoagulable studies
Sedimentation rate, CRP for vasculitis
Blood cultures for endocarditis.

IMAGING
1) Ankle-brachial pressure index:

Brachial pressure is measured with a blood pressure cuff around the arm and a Doppler probe at the
brachial artery – cuff is inflated until the arterial signal is obliterated, then slowly deflated until the
signal just starts being detected, at which the pressure is recorded

Ankle pressures are measured in a similar manner, with the cuff around the calf and the Doppler at
the dorsalis pedis and posterior tibial arteries – one reading for each artery

The ankle pressure to be used for each leg is the higher of the two taken

POST‐GRADUATE GENERAL SURGERY OSCE (By Ali M J) 9


8 LIMB OSCE ‐ Stasions

This ankle pressure is then divided by the brachial pressure (the higher of the two brachial pressures
for both upper limbs) to get the ankle-brachial pressure index

2) Exercise treadmill testing


Measure ABPI before and after patient exercises on a treadmill
If the ABPI falls by >0.2 claudication
3) CXR
4) Doppler US:
Visualizes both venous and arterial systems
Identifies level of arterial occlusion, as well as thrombosis and aneurysm
Sensitivity and specificity >80–90% for occlusion of vessels proximal to the popliteal vessels

5) Plethysmography/segmental pressure measurements:


Uses measurements of the volume and character of blood flow.
Less widely available than US, therefore requires an experienced technician.
Approximates US in sensitivity and specificity.

6) Angiography:
Determines details about the anatomy, including the level of occlusion, stenosis, and collateral flow
Useful where the diagnosis of acute ischemia is uncertain or before emergent bypass grafting.

7) CT angiogram:
CT is useful for diagnosis of occlusive aortic disease or dissection.
Rapidly available and reliable
may not be 1st line for patients with renal insufficiency

8) MRI:
Disadvantages are that MRI is time-consuming and expensive.

DIFFERENTIAL DIAGNOSIS:

1. Acute thrombosis or emboli


2. Arterial dissection
3. Deep venous thrombosis
4. Venous insufficiency
5. Compartment syndrome
6. Buerger disease
7. Spinal stenosis
8. Neuropathy
9. Bursitis
10. Arthritis
11. Reflex sympathetic dystrophy

POST‐GRADUATE GENERAL SURGERY OSCE (By Ali M J) 10


8 LIMB OSCE ‐ Stasions

Definitive treatment options:-


A) Acute Limb ischemia:

Surgical Endovascular
Embolectomy
Endarterectomy Thrombolysis
Bypass grafting Angioplasty
Fasciotomy Stenting
Primary amputation

In general, embolectomy is done for embolic occlusion, while thrombolysis is done for thrombotic
occlusion.

1. Embolectomy
Can be done under LA but still require anaesthetist to monitor patient as he may be quite sick
(e.g. AMI), and hyperkalaemia with cardiac arrhythmia can occur after reperfusion

Involves clamping of the involved artery and making an arterotomy.


A Fogarty balloon catheter is inserted into the artery until distal to the clot, then the balloon is
inflated to trawl the clot out of the artery.
Check for forward-bleeding and back-bleeding of the vessel (i.e. free spontaneous flow from
proximal and distal ends of the artery when unclamped)
Flush with heparinised saline.
Check foot – warm foot with good pulse indicates reperfusion.
Important to monitor ECG for any arrhythmias!.
Closure of arterotomy with meticulous haemostasis as patient is on heparin

Post-op: patient monitored in high-dependency; look out for reperfusion injury


The reperfused muscles become oedematous, increasing pressure in the compartments of the leg, like
compartment syndrome:
Patient complains of calf pain
Unable to dorsiflex ankle as the anterior compartment is affected first
Requires three compartment fasciotomy to release pressure

Need to convert to full warfarin anticoagulation, uptitrating dose until INR 2-2.5 before stopping
heparin (patient at risk of further embolic events)

Discharge patient to anticoagulation clinic for follow-up with warfarin advice

2. Thrombolysis
Angiogram done before thrombolysis to locate occlusion
Thrombolysis catheter inserted into the clot, and the thrombolytic agent is infused.
Patient will be in high-dependency with thrombolytic infusion for 6 hours (~1000-4000 units
per minute).
After 6 hours, redo angiogram to check for residual clot; if some clot remains, adjust catheter
into the clot and infuse for 6 more hours.
After complete lysis of the clot, can do angioplasty

Takes much longer than embolectomy

POST‐GRADUATE GENERAL SURGERY OSCE (By Ali M J) 11


8 LIMB OSCE ‐ Stasions

Thrombolysis may be preferred for embolism in a diseased artery, since it may be difficult to trawl
out the clot in a diffusely stenosed vessel – the clot may get caught on a proximal stenosed segment

Results:
Embolectomy has a 20% mortality, almost full success rate
Thrombolysis has a 10% mortality, only 35% successful

B) Chronic Limb ischemia:

1. Conservative
Smoking cessation.
Exercise training
Exercise at least half to one hour every day
Walk until pain comes, rest 2-3 minutes, and walk again.
Podiatrist to teach foot care.
Aggressive management of hyperlipidemia, HTN, diabetes.
Antiplatelets e.g. aspirin

Intervention (endovascular or surgical)


At least 6 months of conservative treatment first
2. Angioplasty
Stenting usually not done for lower limbs except in aortoiliacs (since stent needs to be placed in a
vessel which is relatively fixed and won’t be kinked/bent by movement)

New method: subintimal angioplasty – if lumen is so occluded that guide wire cannot pass through,
the guidewire is threaded into the subintimal space to create a dissection around the occluded
segment, and this space is then angioplastied to create a channel parallel to the actual lumen for
blood to flow through

3. Bypass grafting
Consider bypass when lesions cannot be treated by angioplasty i.e. lesion extends for long distance
through the vessel and/or no lumen for guide wire to pass through (complete occlusion)

4. Amputation

Indications (3 D’s)
1. Dead (Necrotic tissue).
2. Dangerous (Gangrene, ascending sepsis).
3. Damn nuisance (Non-functional limb; bad smell; pain; constant need to dress wound).

o Level of amputation depends on vascularity of the limb and the indication (e.g. if infected,
need to amputate above level of infection)
o As far as possible try to preserve function of the lower limb
o May require revascularisation interventions before amputation to ensure good healing, or to
enable lower amputation
o Do not simply amputate without ensuring good vascular supply to the surgical site, otherwise
the wound will not heal

POST‐GRADUATE GENERAL SURGERY OSCE (By Ali M J) 12


8 LIMB OSCE ‐ Stasions

Types of Amputations:

1. Toe: most common; usually through proximal phalanx. Must not be performed through the
joint; exposes avascular cartilage and won’t heal.
2. Ray: excision of toe through the metatarsal bone.
3. Transmetatarsal: divided at mid-shaft level. Indicated for infection or gangrene.
4. Midfoot: consider only in patients with correctable or absent ischaemia. Types:
Lisfranc (disarticulation between metatarsal and tarsal bones)
Chopart (disarticulation of the talonavicular and calcaneocuboid joints).
5. Ankle level (Syme and Pirogoff): rarely indicated in vascular practice today.
6. Below-knee (Burgess long posterior flap and skew flap).
7. Through-knee, e.g. Gritti–Stokes: useful if orthopaedic metalware in the femur.
8. Above-knee.
9. Hip disarticulation and hindquarter:
malignant disease
extensive trauma
infection or gangrene
non-healing high above-knee amputation.

WHAT ARE THE POSSIBLE POSTOPERATIVE COMPLICATIONS?

Early
1. Stump haematoma
2. Flap necrosis, infection.
3. Stump trauma from falls.
4. Wound-related pain

Late
1. Neuroma formation.
2. Osteomyelitis.
3. Bony erosion.
4. Ulceration.
5. Ongoing ischaemia.
6. Phantom limb pain.
7. Joint contractures.

POST‐GRADUATE GENERAL SURGERY OSCE (By Ali M J) 13


9 TRAUMA OSCE ‐ Stasions

TRAUMA (HISTORY)

Greet & Introduce yourself. Ask his/her permission


Begin by asking: Name, Age, Occupation, if not mentioned previously

At accident site
The exact time of accident ( history in hours)
How the accident happen (passenger, in the front seat )
Where the patient was hit (which part of the body) – left side of the body –side
collision
Other passengers or pedestrians condition –fine
The condition of patient immediately after accident
Transportation
How the patient was transported
The condition of the patient during transportation
In the hospital ( ER, OR , ward)
The condition of the patient in ER
Over the last night what treatment he received and investigation done-cxr, abdominal
us and blood investigation and received iv fluid
Symptoms of the patient at time of history taking
Any loss of consciousness
Respiratory distress
Abdominal pain
Vomiting and the type of vomitus
The color of the urine
Asking about bowel motion and color of feces
Related history (AMPL)
Allergy
any Medications
Past medical, past surgical (Any Systemic illness?)
Last meal
THANKING the patient.

POST‐GRADUATE GENERAL SURGERY OSCE (By Ali M J) 1


9 TRAUMA OSCE ‐ Stasions

CHEST TUBE (EXAMINATION)

Great and Introduce yourself to the patient, and ask his/her permission.
Positioning and exposure.
Wash/Gel your hands.
Ask (Is it Painful or not? Any other pain region?).

inspection
Site of insertion of chest tube
Tissue surrounding the insertion site ( infection ?)
Under water seal position to the patient
Checking the function of chest tube
Type and amount of fluid drained
Auscultation ( both sides)
Anterior auscultation
Lateral auscultation
Posterior auscultation
Indication for removal
Clinical (stable, breath comfortable)
radiological
THANKING the patient.

POST‐GRADUATE GENERAL SURGERY OSCE (By Ali M J) 2


9 TRAUMA OSCE ‐ Stasions

HEAD INJURY

POST‐GRADUATE GENERAL SURGERY OSCE (By Ali M J) 3


10 COMMUNICATION SKILLS OSCE ‐ Stasions

CONSENT

Greet with (use patient name) & Introduce yourself. Ask his/her permission.
Quiet & calm environment.
Invite to sit, alone or someone else.

Consent : Explaining the


C = Condition and natural history and prognosis of the diseases
O = Options of treatment : no treatment , conservative , surgical , radiotherapy
N = Name of the procedures
And types of anesthesia ( in a simple language the patient can understand )
S = Side effects and complications of treatment : operative , postoperative , anaesthetic
, infection
E = Extra procedures ( blood preparation , drains , NG Tube etc )
N =Names of the operating person and assistants
T = Trial : If the procedure is under trial ( Lumpectomy in breast cancer )
S = Second opinion of other family members may be obtained prior to surgery if the
patient is hesitant about decision.

Postoperative recovery and return to work


Concluding the consent with summary of the whole plan
DURING CONVERSATION
Did the candidate look confident , use simple and a clear language
Was the candidate looking to the patient , kind and showing respect to the patient
THANKING the patient.

POST‐GRADUATE GENERAL SURGERY OSCE (By Ali M J) 1


10 COMMUNICATION SKILLS OSCE ‐ Stasions

BREAKING BAD NEWS (SPIKES)

S; SETTING:
Great and Introduce yourself to the patient, and ask his/her permission.
Privacy
Ask for presence of family members or friends Sit down

P; PERCEPTION:
Ask pt what he know about his condition

I; INVETITION:
Respect pt right to know or not to know other

K; KNOWLEDGE:
Give the bad news in small digestible pieces, give a warning that bad news are coming,
avoiding technical & scientific language.

E; EMPATHY:
Downplay the sensitivity of situation or give more helpful prognosis

S; STRATEGY & SUMMARY:


Give management plans & summarize the information

POST‐GRADUATE GENERAL SURGERY OSCE (By Ali M J) 2


10 COMMUNICATION SKILLS OSCE ‐ Stasions

CONSULTATION (HT)

Beginning of the consultation


Patient's name
Patient's age
Clear consultation destination (which disciplinary is required?)
Greeting the consultant doctor
Consultation details
Duration of hypertension
The drug / drugs used by the patient
The doses of the drugs used
Recording the blood pressure (from the given information)
Any chest pain
Any kind of dyspnoea
Clear and direct aims of the consultation
End of the request
Greeting the consultant doctor
Name and position of the consultation writer
Signature of the consultation writer
Date of consultation

POST‐GRADUATE GENERAL SURGERY OSCE (By Ali M J) 3


11 ORTHO OSCE ‐ Stasions

ELBOW JOINT (EXAMINATION)

Great and Introduce yourself to the patient, and ask his/her permission.
Positioning and exposure.
Wash/Gel your hands.
Ask (Is it Painful or not? Any other pain region?).

LOOK (inspection)
with comparison of both Elbow joints
The arms by the body side and arms fully extended
Look for any swelling or deformity
Compare the carrying angles of both sides
FELL (palpation)
Temp
tenderness over:
Medial & lateral epicondyle.
olecranon process
MOVE
Full flextion
( normally the medial, lateral epicondyles and olecranon form equilateral triangle )
Full extension
( normally the medial , lateral epicondyles and olecranon are in one line )
Pronation and suppination
( during which fell the radial head just distal to the lateral epicindyle)
SPECIAL TESTS
Cozen's test for tennis elbow
(clinched hand, pronated forearm, the patient extend his clinched hand while the
examiner try to flex the hand , pain in the common extensors origin)
Mills's test for tennis elbow
(suppinated forearm , flextion of the wrist ,,, pain in the common extensors origin )
THANKING the patient.

POST‐GRADUATE GENERAL SURGERY OSCE (By Ali M J) 1


11 ORTHO OSCE ‐ Stasions

KNEE JOINT (EXAMINATION)

Great and Introduce yourself to the patient, and ask his/her permission.
Positioning and exposure.
Wash/Gel your hands.
Ask (Is it Painful or not? Any other pain region?).

LOOK (inspection)
( start with the patient standing and exposure of both knees)
Standing patient (any skin or joint shape changes or muscle wasting)
Scar: (injury, arthroscope, meniscectomy)
Popliteal swelling (baker cyst) (popliteal aneurysm)
Ask the patient to walk
Ask the patient to squat
Supine patient (for effusion –horseshoe swelling of the suprapatellar pouch)
FELL (palpation)
Measure the Quadriceps circumference at a set distance (fixed bony point; e.g. 15cm above tibial
tuberosity)
Temperature (septic arthritis, inflammatory arthritis).
Tenderness: Patella:
Knee extension (around patella)
Grind test (move patella up & down)
Clarke’s test (push patella against femur & then extension knee)
Both testes for patellofemoral osteoarthritis (PFOA) & painfull
Palpation of the joint line for local tenderness while knee is flexed 45 degree:
Tibial tuberosity (flat foot, 90° flexed knee)
Patellar ligament
Medial and lateral joint lines.
Popliteal fossa
Effusion:
Patellar hollow test [very small fluid] appear with slight flexion at the medial aspect & disappear with
further flexion
Bulge/Swipe test [small fluid] empty one side and sharply swipe the other and observe the bulge in
side you try to empty it
Cross-fluctuation [moderate fluid] one hand empty the suprapatellar space & other hand empty the
medial aspect and sharply swipe to lateral to observe fluid transmission in the medial
Cruciate ligaments:
Posterior Sag and Posterior drawer (PCL)
Anterior drawer (ACL)
Lachman test (sensitive for ACL)
Collateral ligaments: Standard & Modified tests
MOVE
Asking the patient to extend knees
Asking the patient to flex knees
Passive flexion and extension while palpating the knee for clicking
SPECIAL TESTS
Menisci: McMurray’s test
Complete by examining:
Hip (above) & Ankle (below)
Neurovascular state

POST‐GRADUATE GENERAL SURGERY OSCE (By Ali M J) 2


11 ORTHO OSCE ‐ Stasions

SHULDER JOINT (EXAMINATION)

Great and Introduce yourself to the patient, and ask his/her permission.
Positioning and exposure.
Wash/Gel your hands.
Ask (Is it Painful or not? Any other pain region?).

LOOK (front and behind)


Symmetry (both joints)
Skin (scar, sinus)
Muscles wasting (deltoid, supraspinatus, infraspinatus)
Joints shape; any prominent (SCJ; subluxation or osteoarthritis) (clavicle; old fracture) (ACJ;
subluxation)
FELL (palpation)
Temperature
Tenderness:
BONE: (SCJ, clavicle, ACJ, acromion, coracoid, humeral head, greater tuberosity, scapular spine)
MUSCLES (long head of biceps; biceptal groove with internal & external rotation) (supraspinatus;
under the anterior edge of acromion)
MOVE
Flexion 165° [Both Joints]
Abduction 180° (1st 15°; supraspinatus)( The deltoid muscle then takes over and, at about 90°) [Both
Joints]
Note:
1. Rotator cuff tear lead to difficulty initiate abduction
2. Rotator tendinitis (pain from 60-120°); impingement syndrome.
3. Osteoarthritis ACJ (pain from 140-180°); painful high arc
External rotation 60° (elbow flexed 90°) [frozen shoulder] [Both Joints]
Internal rotation (thumb reach mid-thoracic level from behind; T6)
Power
Deltoid (abduction against resistance; axillary N palsy lead to weak & regimental patch)
serratus anterior (Pushing against wall; long thoracic N palsy, winged scapula)
SPECIAL TESTS
Rotator Cuff
Supraspinatus (resist abduction; full internal rotation, flexed 20° & abduction 20°)
Infraspinatus / Teres minor (resisted ext. rotation, elbow flexed to 90°)
Subscapularis (resisted Int. rotation, elbow flexed to 90°)but not specific to subscapularis
(Gerber’s lift-off test) resist the hand lift off posterior and reach T6
Biceps tendon:
Speed Test: resisted to flexion of shoulder with elbow extended and arm supinated
Yergason’s test: resist supination to arm with flexed elbow
Resist elbow flexion
Complete by examining:
Neck (above) and elbow (below)
Neurovascular state
THANKING the patient.

Rotator cuff muscles: Supraspinatus, Infraspinatus, Subscapularis, Teres minor

POST‐GRADUATE GENERAL SURGERY OSCE (By Ali M J) 3


11 ORTHO OSCE ‐ Stasions

RADIAL NERVE (EXAMINATION)

Great and Introduce yourself to the patient, and ask his/her permission.
Positioning and exposure.
Wash/Gel your hands.
Ask (Is it Painful or not? Any other pain region?).

LOOK (inspection) ---with comparison of both sides


Ask the patient to put their hands behind their heads and look for triceps muscle wasting ( in
cases of high lesions)
Ask the patient to put his forearms close together with full pronation and look for forearm
extensor muscles
Ask the patient to lift his arms and keep hands straight and look for wrist drop
SENSATION
Test sensation on the anatomical snuff box
Test sensation on the back of forearm
MOTOR
Triceps ( the patient extend elbow against resistance)
Brachioradialis (the patient flex his elbow in mid prone position --while the elbow is 90
degree -against resistance )
supinator
Wrist extension against resistence
Finger extensor (ask the patient to keep his finger extended against resistance)
Extensor policis longus ( the patient put his hand palm down on the table and ask him to lift
his thump up against resistance)

POST‐GRADUATE GENERAL SURGERY OSCE (By Ali M J) 4


12 PEDIATRIC OSCE ‐ Stasions

Evaluation and Treatment of Constipation in Infants and


Children
Constipation was defined as a delay or difficulty in defecation, present for 2 or more weeks

Duration
Progression during this time (better, worse)
Related to stool
How many times open bowel per week
When he pass motion Stool Amount:
Few
Large (that can block the toilet)
Nature:
Hard large stool (firm).
thick
very loose (watery)
Stool passed without sensation? If yes → Can also be thick and sticky or dry and flaky
Spontaneously pass motion?.
Straining.
need stimulation (PR, medication)

Symptoms associated with defecation


Any Blood or mucous.
Any distress smell (smells more unpleasant than normal stools)
Pain
posturing: typical straight legged, tiptoed, back arching posture
Anal pain
Poor appetite (is it improves with passage of large stool)
Abdominal distension
Fever
Vomiting

Antenatal and Birth


Any problem during pregnancy
Preterm
Vaginal or CS
Breast feeding
Pass meconium 1st 48 hrs
Reluctant to feeding
well, weight and height within normal limits (during your physical visit)

POST‐GRADUATE GENERAL SURGERY OSCE (By Ali M J) 1


12 PEDIATRIC OSCE ‐ Stasions

Weakness in legs, locomotor delay (such as falling over in a child/young person older
than 1 year).
Changes in infant formula, weaning, insufficient fluid intake
Poor diet and/or insufficient fluid intake.
Relevant Hx
Previous attack
fears and phobias, major change in family,
taking medicines

DDx:
Hirschsprung's disease.
Meconium ileus.
Cystic Fibrosis
Perianal streptococcal infection.
Coeliac disease.
Hypothyroidism.
Idiopathic constipation.

EXAMINATION:
1. Appearance of the skin and anatomical structures of lumbosacral, gluteal, Anal regions
Anal position, patency, Fistulae, bruising, multiple fissures.
Flattening of the gluteal muscles, evidence of sacral agenesis, discoloured skin, naevi
or sinus, hairy patch, lipoma, central pit (dimple that you can't see the bottom of),
scoliosis
2. Abdominal examination (distention, mass, organomegaly).
3. A digital rectal examination.
4. Lower limb neuromuscular examination including tone, strength and Reflexes.

INVESTIGATIONS:
1. Blood Tests for coeliac disease and hypothyroidism (T4, TSH, Ca, IRT, Coeliac Ab)
2. Plain abdominal radiograph
3. Ultrasound
4. Barium enema
5. Endoscopy.
6. Manometry
7. Transit studies.
8. Rectal biopsy

TREATMENT:

Conservative: Surgical:
1. Education (bad toilet habit) for bowel training.
4. Stenosis.
2. Diet
5. Hirschsprung's disease.
3. Medication (laxative)

POST‐GRADUATE GENERAL SURGERY OSCE (By Ali M J) 2


12 PEDIATRIC OSCE ‐ Stasions

CYSTIC FIBROSIS (CF)

Delayed growth
Failure to gain weight normally during childhood
No bowel movements in first 24 to 48 hours of life
Salty-tasting skin

Symptoms related to bowel function may include:


Belly pain from severe constipation
Increased gas, bloating, or a belly that appears swollen (distended)
Nausea and loss of appetite
Stools that are pale or clay colored, foul smelling, have mucus, or that float
Weight loss

Symptoms related to the lungs and sinuses may include:


Coughing or increased mucus in the sinuses or lungs
Fatigue
Nasal congestion caused by nasal polyps
Recurrent episodes of pneumonia. Symptoms in someone with cystic fibrosis include:
Fever
Increased coughing
Increased shortness of breath
Loss of appetite
More sputum

Symptoms that may be noticed later in life:


Infertility (in men)
Repeated inflammation of the pancreas (pancreatitis)
Respiratory symptoms
Clubbed fingers

Exams and Tests

1. Immunoreactive trypsinogen (IRT) test is a standard newborn screening test for CF.
A high level of IRT suggests possible CF and requires further testing.
2. Sweat chloride test is the standard diagnostic test for CF. A high salt level in the
patient's sweat is a sign of the disease.

Other tests that identify problems that can be related to cystic fibrosis include:
1. Chest x-ray or CT scan
2. Fecal fat test
3. Lung function tests
4. Measurement of pancreatic function
5. Secretin stimulation test
6. Trypsin and chymotrypsin in stool
7. Upper GI and small bowel series

POST‐GRADUATE GENERAL SURGERY OSCE (By Ali M J) 3


12 PEDIATRIC OSCE ‐ Stasions

Treatment:
A) for lung problems includes:
1. Antibiotics to prevent and treat lung and sinus infections.
2. Inhaled medicines to help open the airways
3. DNAse enzyme therapy to thin mucus and make it easier to cough up
4. High concentration of salt solutions (hypertonic saline)
5. Flu vaccine and pneumococcal polysaccharide vaccine (PPV) yearly (ask your health
care provider).
6. Lung transplant is an option in some cases.
7. Oxygen therapy may be needed as lung disease gets worse.

B) Lung problems are also treated with aerobic exercise or other therapies to thin the
mucous and make it easier to cough up out of the lungs. These include a Percussion
Vest, manual chest percussion, A-capella, or TheraPEP device.

C) Treatment for bowel and nutritional problems may include:


1. A special diet high in protein and calories for older children and adults
2. Pancreatic enzymes to help absorb fats and protein
3. Vitamin supplements, especially vitamins A, D, E, and K

D) Care and monitoring at home should include:


1. Avoiding smoke, dust, dirt, fumes, household chemicals, fireplace smoke, and mold or
mildew.
2. Drinking plenty of fluids. This is particularly true for infants, children, in hot weather,
when there is diarrhea or loose stools, or during extra physical activity.
3. Exercising two or three times each week. Swimming, jogging, and cycling are good
options.
4. Clearing or bringing up mucus or secretions from the airways. This must be done one to
fours times each day. Patients, families, and caregivers must learn about doing chest
percussion and postural drainage to help keep the airways clear.

Possible Complications
1. The most common complication is chronic respiratory infection.
2. Bowel problems, such as gallstones, intestinal obstruction, and rectal prolapse
3. Coughing up blood
4. Chronic respiratory failure
5. Diabetes
6. Infertility
7. Liver disease or liver failure, pancreatitis, biliary cirrhosis
8. Malnutrition
9. Nasal polyps and sinusitis
10. Osteoporosis and arthritis
11. Pneumonia, recurrent
12. Pneumothorax
13. Right-sided heart failure (cor pulmonale)

POST‐GRADUATE GENERAL SURGERY OSCE (By Ali M J) 4


12 PEDIATRIC OSCE ‐ Stasions

POST‐GRADUATE GENERAL SURGERY OSCE (By Ali M J) 5


12 PEDIATRIC OSCE ‐ Stasions

POST‐GRADUATE GENERAL SURGERY OSCE (By Ali M J) 6


12 PATINET SAFETY OSCE ‐ Stasions

RETAINED SURGICAL SPONGE

Risk Factors:
1. Patient with high BMI
2. Emergence surgery
3. Unplanned change in procedure plan
4. Multiple surgeons involved in the same patient
5. Multiple procedure in the same patient
6. Multiple operation room nurse/staff members.
7. Multiple nurse shift

Decrease the risk by: (no single tool)


1. Use radio-opaque sponge
2. Double counting
3. Inspect the wound before closure.
4. Not use small or non-radiological detectable sponge in large cavities
5. Routine radiography in patient undergo multiple procedure.

Risk factors for wrong side surgery:


1. Bad Communication (error) → 70% (most common cause)
2. Multiple surgeons involved in the same patient
3. Multiple procedure in the same patient
4. Emergence surgery
5. Morbid obesity
6. Time pressure
7. Abnormal patient anatomy
8. Incomplete preoperative assessment (unavailable or not reviewed)
9. Lack teamwork.
10. Surgeon replacement.

POST‐GRADUATE GENERAL SURGERY OSCE (By Ali M J) 1


14 DAY CASE SURGERY OSCE ‐ Stasions

DAY CASE SURGERY

a patient who is admitted for investigation or operation on a planned non-resident basis and who
nonetheless requires facilities for recovery

'out-patient cases': These are minor procedures performed under a local anaesthetic which do not
generally require postoperative recovery time.

Suitability for day case surgery:

physical status - ASA classes I or II are permitted


age
type of surgery
length of anaesthesia
type of anaesthesia
intubation
surgeon
recovery criteria
transport
postoperative pain relief

Type of surgery:
About 50% of all elective procedures were best done on a day case basis:

general surgery:
o hernia repair, e.g. inguinal, femoral, epigastric
o varicose vein surgery
o breast lump excision
o anal stretch
o pilonidal sinus
orthopaedic surgery:
o carpal tunnel release
o release of trigger finger
o Dupuytren's contracture surgery
o arthroscopy
o amputation of finger or toe
o ingrowing toe nails
urological surgery:
o circumcision
o cystoscopy with or without biopsy
o hydrocoele surgery
o excision of epididymal cyst
o reversal of vasectomy
paediatric surgery:
o circumcision
o inguinal herniotomy
o hydrocoele surgery
gynaecological surgery:
o D&C
o termination of pregnancy
o laparoscopy

POST‐GRADUATE GENERAL SURGERY OSCE (By Ali M J) 1


14 DAY CASE SURGERY OSCE ‐ Stasions

ENT surgery:
o myringotomy & insertion of grommets
o direct larygoscopy & pharyngoscopy
o submucous resection
plastic surgery: blepharoplasty, breast augmentation, insertion of tissue expanders

Advantages:

patients:
o know when operation will be, little risk of cancellation
o minimal time away from home which is particularly beneficial for paediatric patients
surgeons:
o less risk of cancellation permits better scheduling of operating lists
o greater turnover of cases
o less delay between cases, usually because less preparation is required
o release of in-patient beds that would have been occupied by day case patients
general practitioners:
o happier patient
o financial saving ranging from 19% to 70% compared to in-patient treatment
hospital management:
o cost-effective treatment, still attaining clinical goals
o facilitates less demand for in-patient beds

disadvantages

the need for a responsible person to oversee the day case patient at home for the first 24-48
hours
the restriction of day case surgery to experienced senior staff; little opportunity for junior
staff to practise
extra work for the general practitioner in the postoperative period; patients often ring them
for advice or treatment
the cost-effectiveness of the unit is reduced when less complex cases are dealt with on a day
basis

POST‐GRADUATE GENERAL SURGERY OSCE (By Ali M J) 2


14 DAY CASE SURGERY OSCE ‐ Stasions

contraindications:

medical:
o angina at rest } all ischaemic
o myocardial infarct in last six months } heart disease is controversial
o hypertensives - diastolic greater than 105 mmHg
o cardiac failure
o acute respiratory infection
o asthma - moderate to severe require increased observation
o chronic bronchitis
o emphysema
o gross obesity: body mass index > 35
o insulin dependent diabetics
o coagulation defects
o anticoagulation therapy
o deformity or anatomical problems in jaw or neck
o extent of pathology, e.g. large scrotal hernia
o operation longer than one hour
o painful operations - need an increase in analgesia

psychological:
o psychologically unstable, e.g. psychosis
o concept of day surgery unacceptable to patient

social:
o lives over one hour away from unit
o no reliable person to drive patient home after surgery and look after them for the first
24-48 hours postoperatively
o at home, no access to a lift, telephone or indoor toilet and bathroom

Investigations should include:

haemoglobin in all females and all males older than 50


electrolytes in all patients older than 70, and in all patients on digoxin, diuretics, and other
cardiovascularly-active drugs
ECG in all patients over 50, and all smokers over 40 years
chest radiology in all patients over 70 years

Preoperative preparation should include:

explaining to the patient the exact nature of the problem and the surgery to be undertaken
the protocol of the day unit
fasting
need for an accompanying person
transport
postoperative restrictions on driving and the use of heavy machinery, HGVs in the
succeeding days
avoidance of alcohol
drug requirements

POST‐GRADUATE GENERAL SURGERY OSCE (By Ali M J) 3


14 DAY CASE SURGERY OSCE ‐ Stasions

Analgesia:

opiates are used less extensively; fentanil and alfentanil are used because of their rapid
metabolism
long-acting local anaesthetic, e.g. bupivacaine, is added to block the region of surgery, e.g.
wound infiltration
mild-to-moderate pain is controlled with paracetamol or dextropropoxyphene
moderate-to-severe pain is countered with NSAID's, e.g. diclofenac, or slow-release
morphine

The following should be considered before discharging patients from day surgery:

no acute surgical complications


no excessive pain
patient awake and orientated
recovery room staff happy about discharge
vital signs stable
patient able to take fluids well
leave with a responsible adult
understand limitations of activity post-anaesthesia
patient passing urine

Complications:

haemorrhage, pain in 50%


nausea and vomiting, dizziness in 50%

POST‐GRADUATE GENERAL SURGERY OSCE (By Ali M J) 4


15 NUTRITIONAL STATUS OSCE ‐ Stasions

ASSESSING NUTRITIONAL STATUS

Wash hands
Introduce self
Permission - ask to examine patient
Expose down to underwear
Reposition lying flat

A) GENERAL INSPECTION:
Around the patient for:
Nutritional supplements.
'Nil by mouth' (NBM) signs.
Patient as a whole
Comfortable?
Is there food at the bedside? Are meals unfinished?
Do they have nutrition supplements on their table?
Do they have a naso-gastric (NG) tube in situ
Do they have 'Total Parenteral Nutrition' (TPN) running - usually a covered plastic bag on a
drip stand, running through a long line (PICC line/Hickman line/central line).
Do they have intravenous fluids running?

B) SPECIFIC INSPECTION:
1. Hands: Nails, for:
Clubbing (could be in keeping with Cystic Fibrosis, Crohns Disease, or causes of cachexia
such as malignancy or tuberculosis).
Koilonychia (iron deficiency).
Leukonychia (low albumin).
Xanthomata (hypercholesterolaemia).

2. Arms, for: Loose skin over upper arms suggestive of rapid weight loss?
3. Face, for:
Hair (Rough and wirey?)
Eyes, for:
- Corneal arcus (hypercholesterolaemia).
- Xanthelasma (hypercholesterolaemia).
- Xerophthalmia - reduced tear formation. May be due to vitamin A deficiency.
4. Mouth:
Angular stomatitis (vitamin deficiency).
Glossitis (vitamin/mineral deficiency).
Apthous ulcers (in keeping with Crohns Disease).
Gums (Gingivitis?)

5. Neck: Goitre? (May be caused by Iodine deficiency).


6. Abdomen:
Thin patient?
Palpate for ascites (which can be due to low albumin states, such as malnutrition or liver
failure)
7. Legs:
Bowing of the legs (low Calcium or Vitamin D).
Pitting leg oedema (due to low albumin)

POST‐GRADUATE GENERAL SURGERY OSCE (By Ali M J) 1


15 NUTRITIONAL STATUS OSCE ‐ Stasions

C) To conclude the examination:


1. Calculate the patient's Body Mass Index = (mass in kg) / (height in square meters).
2. Check the patient's food chart and fluid balance chart.
3. Examine the patient's hydration state.

Thank the patient and offer to help them get dressed.

D) If there is evidence of a specific nutritional deficiency then you could offer to further
investigate as appropriate:
1. Full blood count and haematinic blood tests (Iron, Vitamin B12, Folate).
2. Iron deficiency (depending on other factors): oesophagogastroduodenoscopy (OGD) and
colonoscopy.
3. Vitamin B12 deficiency: Shilling test.
4. Mixed deficiency: Coeliac serology.
5. Bone radiographs or DEXA scan for osteomalacia.

Constituents of a parenteral nutrition diet?


Macronutrients: consisting mainly of energy sources. Many regimens have a combination of
50% carbohydrate and 50% lipid. Nitrogen sources are present as amino acids.
Micronutrients: vitamins, electrolytes and trace elements.

Enteral nutrition:
Conditions:
(1) At least 100 cm of small intestine.
(2) an intact ileocecal valve.
(3) adequate airway protection.

Contraindications:
(1) Gastroparesis.
(2) Intestinal obstruction.
(3) Paralytic ileus.
(4) high-output enteric fistula.
(5) short bowel syndrome.
(6) Severe gastrointestinal bleeding.
(7) No access to the gastrointestinal tract.
(8) Aggressive nutrition not wanted by the patient.
(9) Short-term need for enteral nutrition (<5 to 7 days), 10. Severe malabsorption.
(10) Hemodynamic instability.

Types:
Nasogastric tubes are preferred for short-term feedings (<4 weeks) and can be inserted in the
stomach, duodenum, or jejunum.
Long-term feedings (>4 to 6 weeks) require the placement of a more permanent
gastrointestinal access device: (1) a percutaneous enteral device, (2) a laparoscopically placed
tube (gastrostomy or jejunostomy), or (3) a surgically placed tube (gastrostomy or
jejunostomy).

POST‐GRADUATE GENERAL SURGERY OSCE (By Ali M J) 2


15 NUTRITIONAL STATUS OSCE ‐ Stasions

Methods for administering:


(1) Bolus or gravity, in which 250 to 500 mL of formula is administered quickly several times
per day to patients with relatively normal digestion and absorption.
(2) Intermittent feeding, administered several times per day over a period of at least a half-hour
to allow gastric emptying.
(3) Continuous feeding, initiated at a slow rate (20 to 40 mL/h), and advanced as tolerated.

Complications:
(1) metabolic (e.g., overhydration or underhydration).
(2) gastrointestinal (e.g., diarrhea, nausea, vomiting, delayed gastric emptying, constipation, or
abdominal distention).
(3) mechanical; (e.g., the wrong tube size or a cracked tube)

Note that the typical tube feeding regimen requires additional water to ensure adequate hydration.

Diarrhea may be related to:


(1) factors not associated with the feeding formula.
(2) factors related to the tube feeding formula including too rapid an infusion rate.

The diarrhea may be controlled by:


(1) medications such as Loperamide once Clostridium difficile has been ruled out.
(2) changing to continuous feeding.
(3)slowing the rate of tube feeding.

Types of formulas:
(1) Slandered
(2) Elemental
(3) Concentrated
(4) High protein nitrogen formula
(5) Fiber containing formula
(6) Special formula

POST‐GRADUATE GENERAL SURGERY OSCE (By Ali M J) 3


16 ORGAN DONATION & TRANSPLANT OSCE ‐ Stasions

HOW YOU SELECT AND PREPARE PATIENT FOR ORGAN DONATION:

1. GCS= 3 + Apnea + Exclude (Hs & Ts) reversible cause of cardiac arrest

Hypoxia, hypovolemia, hypothermia, hypoglycemia, hypo-hyperkalemia


Toxin (drugs), tamponade (cardiac), tension pneumothorax, thromboembolism.

2. Confirm death:
Lack of pupil reaction
Lack of corneal reflex
Caloric testing
Lack of gag reflex
Lack of cough reflex
Lack of respiratory effort when taking off a ventillator

Two other senior doctor confirm death (same manner) each alone, or both together.

3. Donation Card or Consent of donation (first kin)


4. Call donation office
5. Exclude donation (contraindication):
Septicemia
Hepatitis B (C for C)
CA (disseminated, Melanoma, Choriocarcinoma)
HIV
Selective organ Failure
CORONA disease
Guru

POST‐GRADUATE GENERAL SURGERY OSCE (By Ali M J) 1


16 ORGAN DONATION & TRANSPLANT OSCE ‐ Stasions

INDICATIONS FOR A LIVER TRANSPLANT


1. Acute liver failure (fulminant hepatic failure).
2. Chronic liver failure.
3. Liver cancer (hepatocellular carcinoma).
4. Metabolic liver disease:
Hemochromatosis.
Wilson's disease.
Alpha-1-antitrypsin deficiency.
Glycogen storage disease.
Tyrosinemia.
Familial amyloidotic polyneuropathy.
5. Other metabolic diseases caused by liver based inborn errors.

Indications for a liver transplant in patients with acute liver failure (sch/rush):
A) Acetaminophen toxicity
PH 7.30
Prothrombin time > 100 s (INR > 6.5)
Serum creatinine >300 mmol/l (> 3.4 mg/dl)

B) No acetaminophen toxicity
Prothrombin time> 100s (INR > 6.5)
age <10 or >40 y
Non A non B hepatitis
Duration of Jaundice before onset of encephalopathy >7 d
Serum creatinine >300 mm/L (>3.4 mg/dl).

POST‐GRADUATE GENERAL SURGERY OSCE (By Ali M J) 2


17 OTHERS OSCE ‐ Stasions

Differential diagnosisLeft iliac fossa (LIF) pain:

May be acute or chronic/subacute.

Gastrointestinal causes:
1. Gastroenteritis
2. Constipation
3. Diverticulitis
4. Volvulus
5. Left inguinal/femoral hernia.
6. Appendicitis: rarely, this can present as LIF pain, particularly in patients with
redundant and loosely attached caecum.
7. Irritable bowel syndrome
8. Carcinoma of rectum or descending colon
9. Crohn's disease and ulcerative colitis (inflammatory bowel disease)

Gynaecological causes
1. Ectopic pregnancy in the left Fallopian tube.
2. Threatened or complete abortion:
3. Causes of LIF pain in later pregnancy: premature labour, placental abruption, uterine
rupture.
4. Pelvic inflammatory disease (PID)/salpingitis/pelvic abscess
5. Mittelschmerz: this is a sudden onset of mid-cycle pain.
6. Ovarian torsion:
7. Fibroid degeneration.
8. Pelvic tumour.
9. Endometriosis

Urological causes
1. Testicular torsion or epididymo-orchitis:
2. Ureteric colic.
3. UTI:

Other causes
1. Abdominal aortic aneurysm (AAA):
2. Situs inversus:
3. Herpes zoster:
4. Pelvic vein thrombosis.
5. Hip pathology.

POST‐GRADUATE GENERAL SURGERY OSCE (By Ali M J) 1

You might also like